Categories
Economic History Exam Questions Harvard Yale

Harvard. Final Examination, U.S. Economic History. Callender, 1899-1900

 

This post is a cross between “get to know an economics Ph.D. alumnus (Harvard)” and a deposit into the data bank of old exams. For three years at the end of the 19th century Guy Stevens Callender taught U.S. economic history at Harvard where he received a Ph.D. in 1897.  He ultimately went on to a professorship at Yale. One of the connections that I discovered in preparing the post is that Guy Stevens Callender and John R. Commons were undergraduate classmates at Oberlin.

For an article about Callender’s contributions:

Engelbourg, Saul. Guy Stevens Callender: A Founding Father of American Economic History. Explorations in Economic History. Vol. 9, 1971-72, pp. 255-267.

_________________

Biographical note:

Guy Stevens Callender was born on 9 November 1865 in Hartsgrove, Ohio, the son of Robert Foster Callender and Lois Winslow Callender.  The family moved from Massachusetts to the Western Reserve when Callender was a child.  At an early age he demonstrated that he had an active mind, intellectual curiosity, and a strong physical constitution; these attributes, along with his being an avid reader of books, led him at the age of fifteen to teach in the district schools of Ashtabula County.  Using his savings from several winters of teaching and his summer earnings made working on the family farm, Callender succeeded in paying for college preparatory courses at New Lyme Institute, South New Lyme, Ohio.

In 1886, at the age of twenty-one, Callender enrolled at Oberlin College where he took the classical course.  There he was influenced by James Monroe, professor of political science and modern history, who taught courses in political economy and sponsored Callender’s volunteer work in the Political Economy Club.  Callender also was an active participant in extracurricular organizations, including the Oberlin Glee Club, Oratorical Association, Phi Delta Society, The Review (student newspaper), and the Traveling Men’s Association.  In these groups, some of Callender’s affinity for leadership and exactness became evident (i.e., service as the financial manager and secretary).  He graduated with the degree of Bachelor of Arts in June, 1891, counting among his classmates John R. Commons and Robert A. Millikan.

After a year spent traveling and working in the business departments of newspapers in Cincinnati, Indianapolis, and Chicago, enrolled (1892) for graduate study at Harvard University from which he received a B.A. (1893), an M.A. (1894), and a Ph.D. in political science (1897).  During his graduate studies at Harvard he served for some time as instructor in economics at Wellesley College, and he was considered an “outstanding man among our graduate students” by Frank W. Taussig and other members of the teaching faculty.  Following the award of his Ph.D., Callender held an appointment as instructor in economics at Harvard from 1897 to 1900.  There he conducted a course in American economic history, which he personally created.  In 1900 he was appointed professor of political economy at Bowdoin College; in 1903 he accepted an appointment as professor in the Sheffield Scientific School of Yale University, where he continued to teach and engage in scholarly research until 1915.  He also served as a member of the Governing Board of the Sheffield Scientific School. In 1904 Callender married Harriet Belle Rice; they had one son (Everett, b. 1905).

Callender published his only book, Selections from the Economic History of the United States, 1765-1860 in 1909.  In it he revealed his entire theory of the progress of the United States from the beginning of colonization until the Civil War.  Callender’s most important contributions are to be found in his condensed, precisely written introductory essays that precede each chapter. His article “The Early Transportation and Banking Enterprises of the States in Relation to the Growth of Corporations,” in the Quarterly Journal of Economics (November 1902) was also well recognized and consulted by scholars.

Callender was as a member of the American Historical Association and the American Economic Association, and he was a frequent contributor as a book reviewer, essayist, and speaker.  Callender’s contribution to scholarship is probably best summed up in his “The Position of American Economic History,” American Historical Review 19 (October, 1913).  Therein he argued that American economic history should “be pursued as a separate subject of study” and that economic historians must be prepared to interpret facts.  For Callender economic history was more than the chronological recital of events of commercial and industrial significance.  He sought historical explanations by applying the principles of economic science to the economic and social development of communities.  His published studies included an analysis of the part played by economic factors in the adoption of the Federal Constitution and in the debate over the economic basis of slavery in the South.

Prior to his death, Callender worked on several writing projects, including a comprehensive, multivolume economic history of the United States, but poor health prohibited him from completing this project.  Another work in progress was a critical essay of Arthur Young’s Political Essays Concerning the British Empire (1772), which focused on the history of British colonies in America.  Until then, Young’s essays had not been generally appreciated or known by American scholars.  Callender was also at work on an introduction for a new edition in two volumes of American Husbandry, which was first published in London in 1775.  Callender’s review of Cyclopedia of American Government (edited by A.S. McLaughlin and Albert Bushnell Hart) appeared in the Yale Reviewshortly after his death.  According to commentator Co Wo Mixter, this highly critical review showed “in a marked degree the range, vitality and acuteness of his thinking” (Yale Alumni Weekly, Oct. 1, 1915, p. 48).

Callender was the recipient of numerous awards and honors.  In 1907 Yale University awarded him an honorary M.A.  Two months before his death the Oberlin College chapter of Phi Beta Kappa elected him to membership.  Upon Callender’s death from a cerebral hemorrhage in Branford, Connecticut, on 8 August 1915, members of the Oberlin College Class of 1891 purchased from his widow his library of some 2500 volumes and gave it to the institution in his memory.  The Class raised additional funds to purchase other titles on economic history, thus rounding out and completing the collection.  A small amount of money was also set aside as an ongoing fund to keep the collection up-to-date.  Callender’s gift to the College Library, established by his graduating class, set an Oberlin precedent.

Source:  Oberlin College Archives.  Guy Stevens Callender Papers, 1820-1870.

_________________

Course Enrollment
1899-1900

[Economics] 6. Dr. [Guy Stevens] Callender.—The Economic History of the United States. Lectures (2 hours); discussions of assigned topics (1 hour); 2 theses.

Total: 163.  11 Graduates, 64 Seniors, 58 Juniors, 19 Sophomores, 11 Others.

Source:  Harvard University. Annual report of the President of Harvard College 1899-1900, p. 69.

_________________

Course Description
1897-98

[Economics] 6. The Economic History of the United States. Tu., Th., at 2.30, and a third hour at the pleasure of the instructors. Mr. Callender.

Course 6 gives a general survey of the economic history of the United States from the formation of the Union to the present time, and considers also the mode in which economic principles are illustrated by the experience so surveyed. A review is made of the financial history of the United States, including Hamilton’s financial system, the second bank of the United States and the banking systems of the period preceding the Civil War, coinage history, the finances of the Civil War, and the banking and currency history of the period since the Civil War. The history of manufacturing industries is taken up in connection with the course of international trade and of tariff legislation, the successive tariffs being followed and their economic effects considered. The land policy of the United States is examined partly in its relation to the growth of population and the inflow of immigrants, and partly in its relation to the history of transportation, including the movement for internal improvements, the beginnings of the railway system, the land grants and subsidies, and the successive bursts of activity in railway building. Comparison will be made from time to time with the contemporary economic history of European countries.

Written work will be required of all students, and a course of reading will be prescribed, and tested by examination. The course is taken advantageously with or after History 13. While an acquaintance with economic principles is not indispensable, students are strongly advised to take the course after having taken Economics 1, or, if this be not easy to arrange, at the same time with that course.

 

Source: Harvard University, Faculty of Arts and Sciences. Division of History and Political Science Comprising the Departments of History and Government and Economics, 1897-98.  pp. 32-33.

_________________

1899-1900
ECONOMICS 6
[Final examination, 1900]

  1. Into what periods may the economic history of the United States be properly divided? Give your reasons for making such a division, pointing out the chief characteristic of each periods.
  2. “A monopoly may be either legal, natural, or industrial.”—
    Distinguish each of these from the others by examples, and explain at length what is the character of an “industrial monopoly.”
  3. What legislation, if any, do you think is needed for the control of trusts? Give in full the reasons for your opinion.
  4. What features of American railway legislation do you consider open to criticism?
  5. “…As has been pointed out in the preceding chapter, cotton culture offered many and great advantages over other crops for the use of slave labor; but slavery had few, if any advantages over free labor for the cultivation of cotton….”—
    (a) Point out some of the advantages of cotton over other crops for the use of slave labor. (b) How do you reconcile the last part of the statement with the fact that cotton was produced chiefly by slave, instead of free, labor?
  6. Considering the conditions prevailing among the negroes in the South as well as in the West Indies since emancipation, what criticism, if any, would you make upon the policy of emancipation as actually carried out by the federal government during and after the war?
  7. What influences can you mention which have contributed to the recent depressed condition of cotton producers? (Do not confine your attention to the “credit system.”)
  8. What were the principal provisions of the resumption act? Explain the conditions under which it was carried into effect.
  9. Explain the conditions which led to the crisis or 1893.
  10. What reasons can you give to support the proposition that immigration has increased the population of the United States but little, if any?

 

Source:  Harvard University Archives.  Harvard University. Final examinations, 1853-2001.Box 2, Folder “Final examinations, 1899-1900”.

Categories
Columbia Economists

Columbia. Economics department in WWII. Excerpt from letter to President Butler, Nov. 1942

 

There is a lot of information packed into the annual budget requests submitted by an economics department. Below I have limited the excerpt from the November 30, 1942 budget submission by the head of the economics department to Columbia President Nicholas Murray Butler to a brief introduction that provides an executive summary of the state of staffing and enrollment one year into the Second World War for the U.S. 

_____________________

Excerpt from R. M. Haig’s Budgetary Requests for 1943-44

DEPARTMENT OF ECONOMICS

File: R. M. Haig
November 30, 1942

President Nicholas Murray Butler
Columbia University

My dear Mr. President:

[…]

Introductory

Before turning to the detailed proposals, it may be helpful to outline certain facts regarding the general situation we face.

  1. The war has made heavy inroads on both our staff and our students. Two ([James W.] Angell and [Arthur R.] Burns) of our ten regular professor offering graduate instruction are in Washington on war service and most of those who remain are devoting a substantial portion of their time to the war effort. The staff of Columbia College has been even more heavily hit. Of the men giving instruction in the college in 1940-1941, [Carl Theodore] Schmidt and [Charles Ashley] Wright are now army officers, [Hubert Frank] Havlik, [Clement Lowell] Harriss, [Walt Whitman] Rostow and [Donald William] O’Connell are in war work in Washington, and [Robert] Valeur is devoting most of his time to aiding the Free French. However, we have been exceedingly fortunate in the substitutes we have been able to secure and (especially as compared with other institutions) we present a strong front in spite of our losses.
  2. It was apparent a year ago that the demand from Washington for persons with graduate training would sweep large numbers of our students from their classrooms before the completion of their courses. Requisitions for economists continue to arrive in almost every mail although we have long since placed in positions everyone on our eligible lists. Yet, as we anticipated last year, our body of graduate students still remains at a figure that makes it desirable and necessary to offer substantially all of our fundamental courses. The decline in the number of our students since our peak year (1938-1939, when 340 were registered) has been very great. However, we still are the largest graduate department of economics in the country by a wide margin. I am told that at Harvard, where there were 115 students last year, only 33 are in attendance this semester, and that at Chicago a similar loss has been suffered. In my letter dated December 30, 1941, it was suggested that we might have as many as 150 graduate students registered in our department this year. The latest count shows 130, with a fair prospect that the figure of 150 will be reached in the Spring session. A poll of the staff shows that, in the opinion of some, the number will be fully as large next year and the consensus is that the number will not be less than 100. Moreover all agree that with the coming of peace we shall be faced with an influx of students which may easily swamp the facilities of our graduate staff. For Columbia College, where this year the enrollment has been large, the outlook for economics in 1942-1943 is very obscure. At this time, it appears probable that the regular offering of courses, at least in skeleton form, will be required to serve a small number of regular students. Fortunately the commitments of the University to individuals on the College staff are such that the situation is highly flexible and can be accommodated with relative ease to whatever special program may be adopted for the undergraduates. In this budget, request is made for appropriations in blank for several instructorships, to be utilized only in case the need for them develops as plans for the college become more definite.
  3. Because of retirements, actual or more or less immediately impending, the department is faced with a serious problem of wise replacement of staff in its graduate division, if we are to maintain in the future the position of eminence we have held in the past. In view of this problem, it has seemed wise to make a virtue of our necessities and to utilize the need for temporary replacements for professors absent on leave during the emergency as an opportunity to invite as visiting professors certain men whom we rate high in the list of possible future staff members. This year we have three such men on the campus ([Oskar] Lange, Arthur F. Burns and [Clarence Arthur] Kulp). We believe that it will be wise to continue this policy of exploration and experimentation next year with funds released from the appropriations for the salaries of [James W.] Angell and Arthur R. Burns, in case the war continues and they do not return to their regular posts.
  4. As an incident to the policy referred to in the preceding paragraph, we have been able this year to offer a remarkably strong series of courses in the field of economic theory. However, we are this year relatively weak in economic history, socialism and industrial organization, offering no courses at all in the last-named subject. The chief embarrassment experienced this year by the unsettled staff situation has been in connection with the supervision of student research. Some of our students who have dissertations in progress have been seriously inconvenienced by the absence of the professors under whom they initiated their studies.

[…]

Source: Columbia University Archives. Central Files 1890-. Box 386, Folder “Haig, Robert Murray 7/1942—1/1943”.

Categories
Harvard Suggested Reading Syllabus

Harvard. Contemporary U.S. Economic History Seminar. Galbraith, 1973

 

 

Not really sure what was actually behind Galbraith giving up his “big course in the Social Sciences” for a cozy post-lunch seminar on Galbraith and the middle-two quarters of the twentieth century U.S. economic history. It seems that you could count the reasons on the middle finger of his right hand. But maybe it reveals nothing more nor less than a desire to simply reduce his teaching obligations to a delightful minimum. Still, not uninteresting to see how John Kenneth Galbraith chose to spend his Wednesday afternoons with a couple dozen Harvard undergraduates nearly a half century ago.

________________

March 16, 1973

Professor and Mrs. R. Paul Levine
Co-Masters, Currier House
Harvard University
Cambridge, Massachusetts 02138

Dear Ursula and Paul:

I’ve given up my big course in the Social Sciences and I’m going to give a smaller seminar in contemporary economic history. Unfortunately there are some reasons why the Department wishes that this be an Economics course—it is something of a problem that, in recent years, my courses have been outside the Department. I wonder, however, if I might schedule it over in Currier House, and I wonder whether, as a further idea, it might be possible to schedule it, say, at 2:00 p.m. on a Wednesday, with the understanding that I would meet beforehand with any students who would like to join me for lunch. I propose to limit the attendance to 20 or 25—always assuming that many want to take it—so the congestion would not be too great. Perhaps you would let me have your thoughts.

Meanwhile my best to you both.

Yours faithfully,

John Kenneth Galbraith

JKG:mjh
cc: James S. Duesenberry

________________

COURSE DESCRIPTION

Economics 2365. Seminar: The United States Since the Great Depression
Professor John Kenneth Galbraith

The Crash and the Slump. The reputable view of cause and cure in the current economic orthodoxy. The collapse of banks, utilities, railroads. The agricultural crisis. Unemployment and the old labor movement. Roosevelt and the rationale of the recovery program. The process of recovery and the impact of Keynes. Radicalism and the rise of the CIO. The approach of World War II. The nature of the wartime economic mobilization. The transition to peace and the rise of economic evangelism. The Fifties and the economics of euphoria. The high tide of the New Economics. The new orthodoxy and the role of conditioned irrelevance.

Half course (fall term). Wednesday, 2-4 p.m.
For Graduates and Qualified Undergraduates. Enrollment limited as necessary.

________________

ECONOMICS 2365
AUTUMN TERM 1973-74
PROFESSOR JOHN KENNETH GALBRAITH

MEETINGS: This course will meet on Wednesday afternoons. Each week at 1:00 there will be an informal lunch in the Currier House private dining room. Class will be from 2:00 to 4:00 in the Currier House Binghem Room. There will be no meeting on Wednesday, November 21st. The course will observe the reading period.

REQUIREMENTS: The major course requirement is a twenty-five page paper due on January 14, 1974. It should develop critically one or another of the subjects discussed in the course. It is expected that the paper will display an understanding of the material presented in class and in the readings; unfamiliarity with relevant lectures and readings, however concealed or explained, will be adversely scored. Each student is to submit the proposed title of his paper by November 21st. Office hours will be arranged in early November for that purpose.

PREREQUISITES: There are no formal prerequisites.

 *  *  *  *  *

I. INTRODUCTION (Sept. 26)

II. THE GREAT CRASH AND ITS CAUSES (Oct. 3)

J. K. Galbraith, The Great Crash 1929

III. THE NATURE OF THE DEPRESSION: DOMESTIC ASPECTS (Oct. 10)

L. Chandler, America’s Great Depression 1929-1941, Chapters 1-7.

IV. THE NATURE OF THE DEPRESSION: WORLD ASPECTS (Oct. 17)

A. Lewis, Economic Survey 1919-1939

V. THE LOGIC OF THE RECOVERY PROGRAM: I (Oct. 24)

A. Schlesinger, The Coming of the New Deal, Chapters 1-10

VI. THE LOGIC OF THE RECOVERY PROGRAM: II (Oct. 31)

A. Schlesinger, The Coming of the New Deal, Chapters 16-25.

VII. THE IMPACT OF KEYNESIAN ECONOMICS (Nov. 7)

J. K. Galbraith, “How Keynes Came to America,” in Economics, Peace, and Laughter. ***
R. Heilbroner, The Worldly Philosophers, Chapter 9—“The Heresies of John Maynard Keynes.”
M. Stewart, Keynes and After, Chapters 4, 6, 11, 12.

VIII: THE NATURE OF WARTIME ECONOMIC MOBILIZATION (Nov. 14)

J. K. Galbraith, A Theory of Price Control
W. K. Hancock, British War Economy, chapters 11, 12, 17 ***

IX. THE NATURE OF WARTIME ECONOMIC MOBILIZATION: THE COMPARATIVE BRITISH AND GERMAN ORGANIZATION (Nov. 28)

B. Klein, Germany’s Economic Preparation for War, (Omit statistical appendix)

X. CRITIQUE OF THE NEW ECONOMICS (Dec. 5)

J. K. Galbraith, The Affluent Society, Chapters 9-25.

XI. THE ECONOMICS OF THE COLD WAR AND VIETNAM (Dec. 12)

P. B. Baran and P. M. Sweezy, Monopoly Capital, Chapter 7—“The Absorption of Surplus: Militarism and Imperialism.”
G. W. Domoff, “Who Made American Foreign Policy 1945-1963.” ***
J. D. Phillips, “Economic Effects of the Cold War.” ***
R. Eisner, “The War and the Economy.” ***

XII. INFLATION AND THE PRESENT CRISIS (Dec. 19)

J. K. Galbraith, “Inflation.”
B. Bosworth, “The Current Inflation: Malign Neglect” in Brookings Papers on Economic Activity, 1973. ***
M. Ulmer, The Welfare State, Chapter 4—“The Anatomy of Inflation and Unemployment.” ***

All of these readings are required. Unless otherwise indicated, the entire book should be read. Readings which are in xeroxed form as well as in book form are marked with a triple asterisk***. Copies of all readings are on reserve in Lamont, Hilles, and Littauer libraries.

 

Source: John F. Kennedy Presidential Library. John Kenneth Galbraith Personal Papers.  Series 5. Harvard University File, 1949-1990. Box 522, Folder “Economics 294: Spring term, 1968 (2 of 2) [sic]”.

Image Source: John F. Kennedy Presidential Library. Biographical Profile: John Kenneth Galbraith.

Categories
Suggested Reading Swarthmore Syllabus

Swarthmore. Honors Economic Theory Seminar. Stolper, ca. 1944

 

This seminar reading list and reports was probably typed by Wolfgang Stolper himself and given to a (the?) departmental secretary for preparing 25 mimeographed copies to distribute. While this typed seminar outline has no date, at least judging from the last item to be reported on, William Beveridge’s Full Employment in a Free Society, the outline probably dates from the 1944-45 academic year.

The course description remained the same throughout Stolper’s years at Swarthmore.

Paul Samuelson was an honors examiner in 1943.

Richard Musgrave also served as an external examiner in 1946.

______________________

Course Description
(taught by Stolper 1941-42 through 1946-47, 1948-49)

The following seminars prepare for examinations for a degree with Honors:

[…]

  1. Economic Theory. Mr Stolper. Each semester.

An advanced analysis of the processes by which the prices of goods and services and the incomes of the factors of production are determined under various market condition. A study of theories of the business cycle. Directed reading and discussion, supplemented by reports on the theoretical problems raised by factual studies.

Source:  Swarthmore College Catalogue, 1941-42 p. 62.

______________________

Stolper
Economics Theory Seminar
25 copies

(1) First Week

Assigned:

Joan Robinson, Imperfect Competition, Bk. I
E.H. Chamberlin, Monopolistic Competition, Chs. 1,2
Ricardo, Principles, Ch. 1
Wicksell, Lectures, Vol. I, Introd. und Pt. I, Sec 1

Suggested:

J.St. Mill, Principles, Bk. III, except Ch. 5
J. Cassels, A Re-interpretation of Ricardo on Value, QJE, Vol. 49, p. 518

Topic:

Review of the Tools of Analysis

(2) Second Week

Assigned:

Wicksell, Lectures, Vol. I, Pt. I, pp. 29-100.  You may omit: p. 31 small print; p. 60-63 small print; p. 79, 2d last para- p. 81 top; p. 93-95, small print and diagram
J. Viner, Cost Curves and Supply curves
Marshall, Principles, Bk. V, Chs. 1-5, 8, 9, 13, 15

Topic:

Pure Competition. The Classical Statement of Value Theory

(3) Third Week

Assigned:

Marshall, op.cit. Bk. V., Chs. 12, 14
Chamberlin, op.cit., Chs. 3-6
Triffin, Monopolistic Competition and General Equilibrium Theory, Pt. I, secs. 1, 2,4; Pt. II complete

Suggested:

Triffin, op.cit. Pt. I, Secs. 3, 5, note

Topic:

Monopolistic Competition

Report:

TNEC Monograph 21, Competition and Monopoly in American Industry, by Clair Wilcox

(4) Fourth Week

Assigned:

Triffin, op.cit., Pt. III, secs. 1, 3, 5, 6, 7

Suggested:

Triffin, Monopoly in Particular Equilibrium…,Econometrica. Vol. IX, No. 2, p. 121
J. Robinson, What is Pure Competition? QJE Vol. 49, p. 104
F.Y. Edgeworth, Pure Theory of Monopoly
Cournot, Mathematical Theory of Wealth

Topic:

Criticisms of Monopolistic Competition

Reports:

(a) The Problem of Excess Capacity(See Bibliography in Chamberlin, op.cit.)
(b) Price Discrimination between Markets

G. Haberler, International Trade, Ch. on Dumping
J. Robinson, op.cit., Ch. XV
For illustrative facts refer to the following:
TNEC Monog. 41, Price Discrimination in Steel
TNEC Monog. 42, The Basing Point Problem

(c) Price Policy

TNEC Monog. 1, Price Behavior and Business Policy
Hall and Hitch, Oxford Economic Papers No. 2, pp. 12-45
Clive Saxton, The Economics of Price Determination
National Bureau of Economic Research, Cost Behavior and Price Policy

(d) F.P. Bishop, The Economics of Advertising

 (5) Fifth Week

Assigned:

Marshall, Bk. VI, Chs. 1, 2, 11
Wicksell, Pt. II, Sec. 1. You may omit: pp. 127-129, small print. Don’t worry about the mathematics on pp. 127-131, 139-40
Meyers, Elements of Modern Economics, Ch. XIII (old edition, Ch. XI)
J. Cassels, The Law of Variable Proportions, Explorations in Economics, pp. 223-228
Chamberlin, op.cit. Ch. VIII (also in Explorations, …., pp. 237-250)
Triffin, Pt. III, Sec. 2
Schumpeter, The Instability of Capitalism, Economic Journal, Vol 38 (1928)

Suggested:

Articles by Machlup, Hart, Smithies, and remainder of article of Cassels, in Explorations in Economics
G. Stigler, Production and Distribution Theories

 (6) Sixth Week

Assigned:

Marshall, Bk. VI, Chs. 3, 4, 5 Bk. IV, Ch. 4, 5
Ricardo, Ch. 5
Hicks, Theory of Wages, Complete

Suggested:

A. C. Pigou, Theory of Unemployment
J.T.Dunlop, Wage Determination under Trade Unions

Topic:

Wage Theory and Wage Problems

Reports:

(a) Colin Clark, The Conditions of Economic Progress, particularly, Chs. V-IX
(b) National Resources Planning Board, Problems of a Changing Population, pp. 1-138
Reddaway, Economics of a Declining Population
Malthus, Principles,
Myrdal, Population
Hansen, Economic Progress and Declining Population Growth, Readings in Business Cycle Theory, pp. 366-85

(c) P. Douglas, Theory of Wages
National Income of the US, Chs. 1-4

(d) TNEC Monog. 22, Technology in our Economy
TNEC Hearing on Technological Progress, selections
Consult for further readings

(e) J.T. Dunlop, op, cit., Chs. I-VI, IX, X
(f) TNEC Monog. 5, Industrial Wage Rates, Labor Costs and Price Policies

(7) Seventh Week

Assigned:

Ricardo, Ch. 2, 3
Mill, Bk. III, Ch. 5
Marshall, Bk. V, Ch. 10, 11, Bk. VI, Chs. o,10
Chamberlin, op.cit. Appendix D

Report:

H.W. Singer, Index of Urban Land Rents, Econometrica, Vol IX,

(8) Eighth Week

Assigned:

Schumpeter, Theory of Economic Development, Chs. 2, 3, 4
Triffin, op.cit., Pt. V, omitting B
Marshall, Bk, VI, Chs. 7, 8
R.A.Gordon, Explorations in Economics, pp. 306-317
Schumpeter, Development, Ch. 1
Wicksell, Pt. II, sec. 3

Reports:

(a) Berle and Means, The Modern Corporation and Private Property
TNEC Monograph 29, The Distribution of Ownership in the 200 largest non-financial Corporations
(b) W.L.Crum, Corporate Size and Earning Power

Topic:

Profits. General Summary of Value and Distribution

(9) Ninth Week

Assigned:

Wicksell, Pt. II, Sec. 2; Pt. III. You may omit: Sec D, pp. 172-184; pp. 203-5, small print; p. 216, 2d para—p. 217, 1stpara
Marshall, Bk, VI, Ch. 6
Schumpeter, Development, Ch. V, I
Keynes, General Theory, Chs. 13, 14
Higgins and Musgrave, Deficit Financing—The Case Examined, Public Policy Yearbook II, pp. 136-207
O. Lange, The Rate of Interest and the Optimum Propensity to Consume, Readings in Business Cycle Theory, No. 8

Suggested:

J.M. Keynes, General Theory
Readings…., Pt. II

Topic:

Interest Theory

Reports:

(a) E.v. Böhm-Bawerk, Positive Theory of Capital, Bks. I-III
(b) E.v. Böhm-Bawerk, Positive Theory, Bks. IV to end
(c) F.A.Lutz, The Structure of Interest Rates, Q.J.E., Vol. 55,

(10) Tenth Week

Assigned:

Haberler, Prosperity and Depression, 1939 ed. or later, Chs. 8, 9, 13
Hicks, Value and Capital, Ch. 24
Hicks, Social Framework, Chs. XI-XVI, Appendices E and F
and either
Schumpeter, Business Cycles, Ch. 4
Kuznets, Review of Schumpeter, AER
or
Mitchell, Business Cycles, Ch. III

Suggested:

Readings…., Pt. I

Topic:

Measurement and Separation of Cycles

Reports:

(a) Burns, Production Trends in the US since 1870
(b) Schumpeter, Business Cycles, Chs. 6, 7
Kondratieff, Readings…, pp. 20-42
(c) Schumpeter, Cycles, Vol. II, Chs. 14, 15
(d) The Cob Web Theorem

(11) Eleventh Week

Assigned:

Schumpeter, Development, Ch. 6
Haberler, Prosperity and Depression, Chs. 1, 2, 3, 5, 8, 13
P.A.Samuelson, Readings…. pp. 261-289

Suggested:

Readings, Pt. III, IV, V.
Tinbergen, Critical Remarks on Some Business Cycle Theories, Econometrica, Vol. 10, pp. 129 ff

Reports:

(a) A.H.Hansen, Fiscal Policy and Business Cycles, particularly Pts. I, III, IV
P.A.Samuelson, A Synthesis of the Principle of Acceleration and the Multiplier, JPE, 1939
_______________, Fiscal Policy and Income Determination QJE, August 1942
(b) Haberler, Prosperity and Depression, Chs. 4, 6, 7
Hansen, Business Cycle Theory
Readings, …., Pts. IV, V
(c) Wm. Beveridge, Full Employment in a Free Society

Topic:

Business Cycles, Theory and Theories

 

Source: Duke University. David M. Rubenstein Rare Book & Manuscript Library. Economist Papers’ Archive. Wolfgang F. Stolper Papers, Box 19, Folder “S miscellaneous (2 of 3) Swarthmore Theory outline”.

Image Source: Wolfgang F. Stolper from  John Simon Guggenheim Memorial Foundation (Fellow, 1947).

Categories
Cambridge Exam Questions

Cambridge. Examination Questions of the Economics Tripos. 1932

 

In the U.S. Library of Congress I came across a collection of the Cambridge University Economics Tripos examinations for 1931-1933. In an earlier post I provided transcriptions of the 1931 exams. This post provides the 21 examinations for 1932. For a later post I’ll transcribe the 1933 exams.

___________________

PART I.

Monday, May 30, 1932. 9—12.
GENERAL PRINCIPLES I.

  1. Bring out clearly the economic principles which determine the relative values of a bushel of wheat and a pair of boots.
  2. Define the following and consider the relations between them: wealth, capital, land.
  3. What do you understand by (a) saving; (b) investment? Explain the process by which savings are converted into capital.
  4. A tenant farmer is cultivating a dairy farm in the West of England and paying a rent of £200 per annum. The landlord dies and the farm is put up for sale and offered for £4000 to the tenant, in the first instance. What various economic factors should influence the farmer in deciding whether to buy or not?
  5. What forces tend to remove and what forces tend to perpetuate inequalities in wage rates (a) in the same industry in different parts of the country; (b) in different industries in the same country?
  6. Bring out the importance for the theory of value of (a) marginal utility; (b) the principle of substitution.
  7. How do you account fort he observed tendency for the prices of foodstuffs and raw materials to fluctuate much more widely than the prices of most manufactured articles?
  8. Under what circumstances is it likely that unrestricted competition will lead to the formation of a monopoly?
  9. On what does the marginal net product of labour depend?
  10. Trace the stages by which a sudden but enduring fall of 50 per cent. in the demand schedule for an article will react upon (a) the supply of that article; (b) its price.
  11. Analyse profits on capital and discuss the tendency of each element to rise or fall.
  12. “To use the phrase ‘negative quasi-rent’ is to misconceive the nature of Marshall’s doctrine of ‘quasi-rent.’” Comment on this statement.

 

Monday, May 30, 19932. 1½ — 4½.
SOCIAL PROBLEMS.

  1. What do you understand by (a) primary poverty, (b) secondary poverty? Are there in your view any immediate means of removing the causes or remedying the effects of primary poverty?
  2. Which is the more suitable body for applying the “means test” to applicants for unemployment benefit, the Ministry of Labour or the Public Assistance authority? Is the test desirable as a permanent feature of the unemployment insurance system?
  3. In which industries is unemployment most severe at the present time? In the event of a general revival of trade in England in the near future, in which industries would you expect employment to increase most rapidly?
  4. Compare with reference to the course of events since the War the relative efficacy of (a) “direct action,” and (b) political pressure for increasing taxation and social services, as methods of improving the welfare of the working classes.
  5. There has recently been a widespread substitution of piece-work for time-work in Russia, leading to large increases of output. Would you regard this as evidence in favour of extending piece-work in other countries?
  6. Consider the arguments for and against the extension of unemployment insurance to cover agricultural workers.
  7. What evidence is there that the population of Great Britain will begin to decline within twenty years? Would a declining population solve (a) the housing problem, (b) unemployment?
  8. Would you favour the use of a Government housing subsidy to provide rent rebates varying according to the size of the family housed?
  9. Under what conditions will the general introduction of the automatic loom in Lancashire aggravate unemployment? Is it desirable to introduce labour-saving machinery during depression?
  10. How would you account for the growing tendency in recent years for the formation of Industrial Unions rather than Craft Unions?
  11. “It is a direct corollary of the ‘marginal productivity’ theory of wages that high wages cause unemployment.” Discuss.
  12. Why are the representatives of organized labour as well as of employers in this country opposed to compulsory arbitration as a method of settling trade disputes?

 

Tuesday, May 31, 1932. 9 — 12.
GENERAL PRINCIPLES II.

A.

  1. How is elasticity of demand measured? Under what conditions is the demand for some product likely to be very elastic?
  2. What evidence would you require in order to discover whether this country is over-populated?
  3. “Price is equal to marginal cost.” Explain the meaning of the word “marginal” in Economics. What is the relation of marginal to average cost?
  4. What are the chief assumptions made in the construction of the competitive theory of value? Are they closely related to the actual conditions of modern economic life?
  5. Show, with a diagram if possible, how a publisher would determine the price that he should charge for a new book.
  6. “Owing to the falling off of sales in 1931, and the consequent increase of overhead cost per unit of output, it has proved necessary for us to advance the prices of our products.” Examine the validity of this argument as applied to a short period.

B.

  1. By what means can a central bank control the level of prices in a country?
  2. What effect on prices would you expect (a) if the practice of paying wages by cheque became more common, (b) if wages were paid monthly instead of weekly?
  3. What is meant by the “purchasing power parity” theory of exchanges? How far do you consider it possible to calculate the present equilibrium rate of exchange between the pound and the dollar?
  4. Can two countries both gain by the existence of a trade between them?
  5. Explain in detail how a Bill of Exchange serves to make payments between persons in different countries.
  6. Give some account of the legal enactments at present governing the issue of paper money in this country, and of the more important changes which have been made in the last hundred years.

 

Tuesday, May 31, 1932. 1½ — 4½.
ESSAY SUBJECTS.

Write an essay on one of the following subjects:

  1. Mr MacQuedy: “Then, sir, I presume you set no value on the right principles of rent, profit, wages and currency?”
    The Rev. Dr. Folliott: “My principles, sir, in these things are, to take as much as I can get, and to pay no more than I can help. These are every man’s principles, whether they be the right principles or not. There, sir, is political economy in a nutshell.”
    (T. L. Peacock: Crotchet Castle.)
  2. The future of party government.
  3. The British Empire as an economic unit.
  4. High Finance.
  5. Sweepstakes.

 

Wednesday, June 1, 1932. 9 — 12.
ENGLISH ECONOMIC HISTORY

  1. Give some account of the “old colonial system” and estimate its importance in the economic development of England from 1660 to 1776.
  2. Examine the causes and the principal consequences of the enclosure movement of the eighteenth and early nineteenth centuries.
  3. “So far from originating cruelty to children, the factory system called attention to the evil by concentrating it where all could see.” Discuss this view.
  4. Compare the outlook and objectives of the Chartists with those of the advocates of the Repeal of the Corn Laws.
  5. What were the principal reasons for the industrial leadership of Great Britain in the ‘fifties and ‘sixties of last century?
  6. “All through the nineteenth century the railways have been the great factor making for the extension of the sphere of State action and the abandonment of the idea of free competition.” Discuss.
  7. Outline the development of direct taxation from Pitt to Gladstone.
  8. Describe and account for the main changes that took place in the general level of gold prices between 1850 and 1914.
  9. What were the principal changes in agricultural organization and policy from the acute depression of the ‘seventies and ‘eighties to the outbreak of the World War?
  10. Give some account of the growth of Trade Unionism and the Labour Movement between 1867 and 1913.
  11. Why did the Protectionist campaign of Mr. Joseph Chamberlain fail, and that of Mr Neville Chamberlain succeed?

 

Wednesday, June 1, 1932. 1½ — 4½.
ECONOMIC STRUCTURE

  1. Give some account of the technical advantages of large scale production in any one industry with whose conditions you are acquainted.
  2. What is meant by “the external economies” of an industry? What difficulties are involved in this conception?
  3. How would you account for the fact that some industries are much more localized than others? In what ways do you consider it likely that the development of the cheaper transmission of electricity will alter the location of British Industries?
  4. Give some account of the objects and methods of Scientific Management.
  5. If in any firm the most efficient technical size of several processes were too large for efficient management, how could the organization be adjusted to minimize this loss of efficiency?
  6. What are the proper functions of advertisement? Do you consider that the total expenditure on advertising in this country is excessive?
  7. How would you explain the fact that in many districts almost every firm turns out more than one product, instead of specializing on a single product?
  8. Describe the more important channels whereby saving is placed at the disposal of industry. What suggestions would you make for their improvement?
  9. Which of the various arguments used to justify the imposition of tariffs at the present moment do you consider valid?
  10. Give an account of the monopoly organisations in any one important British Industry. What factors limit the power of the monopolist to raise prices in the case that you are considering?
  11. Describe the methods employed for the marketing of either cotton or wool at each stage from the raw material to the finished product. Explain why each particular method is adopted, and consider whether any change in the method is desirable.

 

PART II.

Monday, May 30, 19932. 1½ — 4½.
SUBJECTS FOR AN ESSAY.
(OLD AND NEW REGULATIONS.)

  1. Thrift.
  2. The Future of the Party System.
  3. Sky-writing.
  4. The Scope and Method of Realistic Economics.
  5. Disarmament as a Practical Problem.
  6. Man in the Machine Age.

 

Monday, May 30, 1932. 9—12.
MONEY, CREDIT AND PRICES.
(OLD REGULATIONS.)

  1. Do you consider that the legal regulation of the Central Bank’s minimum gold reserve serves a useful purpose in a gold standard country? What form of regulation, if any, do you recommend?
  2. Are there good reasons for holding that the “rate of saving” and “rate of investment” should appear as terms in the equation used to demonstrate the principal factors which govern the value of money?
  3. Explain the working of a forward foreign exchange market. How far can it obviate the inconveniences which arise when there is no fixed par of exchange between currencies?
  4. If it had been decided to stabilize the purchasing power of the national standard of value in terms of an index number of prices, what main categories of goods and/or services would you include in the index number?
  5. What considerations have to be taken into account in analyzing the causes which determine the velocity of circulation of money?
  6. If the principal nations of the world decide to adhere to a gold standard in future, would you consider that the impeding gold shortage gives ground for alarm? State your reasons.
  7. In what respects, if any, do you conceive that the policy of the Federal Reserve System has been open to criticism in the period from 1926 to the present day?
  8. “The resistance of wage earners to reductions in money wages has been of value in saving us from the worst excesses of deflation.” Examine this contention.
  9. On what lines should the Indian currency system be developed?
  10. Do you agree with the view that public works designed to stimulate employment in a slump “are a mere piece of ritual, achieving nothing which could not equally well be achieved b the banking system acting alone, through a sufficiently great alteration in tis terms of lending”?
  11. Would the formation of a large “sterling area” be of advantage or disadvantage to this country, in your opinion, if she were endeavouring to maintain a stable currency divorced from gold?

 

Monday, May 30, 1932. 9—12.
POLITICAL THEORY.
(OLD REGULATIONS.)

 

  1. “The State may be defined as a juridically organized nation.” Discuss this definition.
  2. Would you agree that political obligation is equally binding on the citizen whether he lives in a democracy or under a dictatorship?
  3. Dicey contended that democracy and “collectivism” were inconsistent. Do you accept his contention?
  4. Would you consider that the conception of a right of property as belonging to the individual has permanent value, or would you regard it as only characteristic of a particular phase of social development?
  5. In what sense can we speak of law as being “enacted,” and what are the organs of such enactment in the modern State?
  6. How would you analyse the conception of “public opinion,” and by what methods would you suggest that such opinion should be brought to bear on political government?
  7. On what principles, and by what methods, would you impose limits (if any) on liberty of the expression of thought?
  8. Discuss the prison as one of the institutions of political life, with reference to (a) the purpose which it should serve, and (b) the methods which it should employ.
  9. It has been said that “the process of discovering the Sovereign is in all modern States the same.” By what process would you seek to discover the Sovereign in the modern State?
  10. “The State properly intervenes not to conduct the economic business of the country, but to uphold social standards.” Discuss the value of the distinction here suggested.
  11. ”The problem of our days is not the Man versus the State, as it was when Herbert Spencer wrote in 1884, but the State versus the Group.” Comment on this statement, explaining the sense which you would attach to the term “Group.”

 

Tuesday, May 31, 1932.  1½ — 4½.
DISTRIBUTION AND LABOUR.
(OLD REGULATIONS.)

  1. “Every factor of production tends to be remunerated at a rate equivalent to its marginal net product of commodities in general.” Does this statement need any modifications or qualifications when it is applied to land which are not equally necessary when it is applied to the other factors? Give your reasons.
  2. “The Dole has kept up wages above their proper level.” What are the arguments in support of this view? Discuss their soundness. What changes in public policy, if any, do your conclusions suggest as desirable?
  3. What principal factors determine the magnitude of the change which a fall in the general level of commodity prices occasions in the proportion of the national income accruing to the owners of fixed-interest bearing investments?
  4. With the object of creating employment, a grant is made by the exchequer to a local authority for a road-widening scheme. The local authority sets about doing the work as efficiently as possible with the aid of steam navies and other labour-saving machinery. Ought the Exchequer to challenge this actions? Argue the case for and against.
  5. “The money now paid in unemployment benefits should be used to provide employment.” Suggest possible schemes and discuss their soundness.
  6. Compare the relative levels of wages in skilled and unskilled occupations in this country to-day and before the War. Discuss the bearing of any changes upon (a) the supply of skilled workers, (b) industrial contentment, (c) labour organization and policy.
  7. What are the principal differences between the English and German provisions for State action in the regulation of wages and the settlement of industrial disputes? Discuss whether England could with advantage follow the German example in these matters more closely.
  8. Discuss the relative merits of systems under which wage-rates vary with (a) the market price of the product, (b) a cost of living index, (c) a wholesale price index, (d) the profit of the undertaking.
  9. Explain and discuss the place and function of the Trades Union Congress and the General Council of the Trades Union Congress in labour organization.
  10. If you had to measure the change in the general level of money wages in this country since 1900, what difficulties, theoretical and practical, would you encounter?
  11. If a British lace manufacturer proposed to run his establishment continuously on a three-shift system, what procedure would he have to adopt (a) to conform with legal requirements and (b) to meet possible opposition from any quarter?
  12. What are the functions of the Public Assistance Committees? Do you consider that any of them should be transferred to other authorities or modified in any way? State your reasons.

 

Tuesday, May 31, 1932. 1½ — 4½.
STRUCTURE AND METHODS OF GOVERNMENT IN THE MODERN WORLD.
(OLD REGULATIONS.)

  1. Discuss the powers of the Supreme Court, under the American system of government, in disallowing legislation. Would it, in your view, be possible or desirable to institute a judicial body with similar powers in a nonfederal State?
  2. Discuss the methods by which the House of Commons controls finance. Would you regard those methods as adequate, at the present time, for the purpose of regulating the distribution of expenditure or of enforcing general economy?
  3. Describe the effect of the main changes which have taken place in the relations of the component parts of the British Commonwealth between the outbreak of war in 1914 and the passing of the Statute of Westminster in 1931.
  4. Describe the position and powers of committees of the legislature under the French system of parliamentary government. In what ways do they differentiate that system from the system in Great Britain?
  5. What contrast would you draw between the British party system and that of Germany in regard to (a) their influence on the electorate, and (b) their effects on general working of the Constitution?
  6. How far has England developed the French system of “administrative justice” during the present century? What are the defects of administrative jurisdiction as it exists in England to-day?
  7. What would you regard as the main reforms effected in English local government since 1902, and what further reforms would you advocate in regard to the areas or functions of local authorities?
  8. Describe in general terms the composition and functions of the National Economic Council of Germany, and discuss the tendencies in Great Britain towards the development of a similar system since 1918.
  9. Give some account either of the merits and disadvantages of the system of “indirect rule” in the British Colonial Empire or of the varieties and the working of the system of British Mandated Territories.
  10. Would you agree that there is an increasing recognition in Great Britain of the part which the “Expert” should play in the work of government?
  11. Compare the position of the American Senate with that of the German Reichsrath. How would you account for the comparative weakness of the latter?

 

Wednesday, June 1, 1932. 9 — 12.
STRUCTURE AND PROBLEMS OF INDUSTRY.
(OLD REGULATIONS.)

  1. “The entrepreneur has passed away with the Victorian era. His place has been taken by a combination of financier and salaried manager.” Discuss the truth of this contention.
  2. Discuss the possibility of defining “an industry” so as to make it a useful conception in the analysis of economic structure without doing violence to its popular meaning.
  3. Are there any special circumstances in England to-day that make the rational planning of each industry of more importance than in the past? Illustrate what is included under planning from your knowledge of any one industry.
  4. Explain the nature and causes of any differences that occur in the marketing of (a) producers’ goods, (b) consumers’ goods, and the effect of these differences upon the location of the plants making these two types of goods.
  5. How far can statistical proof be adduced for the contention that some manufacturing industries are subject to the law of increasing returns?
  6. What precisely are the supposed defects in a system of private enterprise that have led to schemes for the public control of such activities as transport, generation of electricity and the supply of water?
  7. “Co-operative societies in agriculture are nothing more nor less than cartels, and threaten the consumer with the same sort of exploitation.” Comment.
  8. What forms of integration and combination seem to you to best calculated to reduce the risks of industrial fluctuations, and why?
  9. Discuss the effect on national and international localization of industry of the modern and the possible future development of (a) the telephone, (b) road transport, (c) commercial aviation.
  10. Are we bound to expect a contraction in British industries manufacturing for export? What statistical sources and methods would you use to estimate the extent of such contraction, and to indicate industries and services to which investment might advantageously be directed or transferred?
  11. Discuss any differences between British and foreign systems of education and training in respect of their effect upon the costs of industrial leadership at home and abroad.

 

Tuesday, May 31, 1932. 9 — 12.
ECONOMIC PRINCIPLES.
(OLD AND NEW REGULATIONS.)

  1. Use “the famous fiction of the ‘stationary State’” to illustrate the uses and abuses of abstraction in the treatment of economic problems.
  2. Analyse, and illustrate by examples, the various ways in which a change in the supply of one commodity may affect the demand for others.
  3. “Under conditions of simple competition in a perfect market, the price of a commodity must, in the long run, be equal both to the marginal and to the average cost of producing it in a representative firm.”
    Explain precisely, what you take this statement to mean, and briefly discuss its validity.
  4. What effect might a substantial tax upon imports of wheat and wheaten flour be expected to have, in this country, upon (a) the price of bread, (b) farming profits, (c) agricultural rents? Give reasons for your answer.
  5. What functions does a company promoter perform, and how are his earnings determined? Frame your answer so as to show how it fits into your general theory of distribution.
  6. In a country like Great Britain, how would an increase in the supply of capital affect the earnings of labour? What do you understand by “an increase in the supply of capital”?
  7. In what circumstances will a monopolist charge different customers different prices for the same product?
  8. How would you proceed if you were required to decide whether profits in any given industry were ‘normal”?
  9. For what reasons may a country permanently import part of its supply of a commodity and produce part at home; and what determines, in such a case, the proportion of the total supply which is imported?
  10. When several different things are produced by the same firm, on what economic principle, if any, is it possible to assign a separate cost of production to each of them?
  11. If the habit of keeping a banking account were to spread among wage-earners in this country, how would the general level of prices be affected?

 

Wednesday, June 1, 1932. 9 — 12.
INTERNATIONAL LAW.
(OLD AND NEW REGULATIONS.)

  1. Discuss the modern theories of the basis of obligation in International Law.
  2. What is the present position in International Law of the right of navigation of international rivers?
  3. What are the sanctions provided by the Covenant of the League of Nations for its enforcement? They have been termed “Pseudo-Sanctions”; do you consider there is any justification for this?
  4. Under what circumstances is a war-ship justified in interfering with a merchant ship of another State on the high seas in time of peace?
  5. What is the connection between the Permanent Court of International Justice and the League of Nations?
    What is implied by signing the “Optional Clause”?
  6. Explain the circumstances under which a State may (a) sue, (b) be sued in a foreign Court of Law.
  7. Article 21 of the Covenant of the League of Nations speaks of “regional undertakings like the Monroe Doctrine for securing the maintenance of peace.”
    Comment on this.
  8. Give the provisions of the Kellogg-Briand Pact, 1928. What methods exist for the pacific settlement of international disputes?
  9. What is the position of
    (a) Sea-borne Mails,
    (b) Submarine Cables,
    in time of war?
  10. What were the methods employed by the Allied Powers in the restriction of enemy commerce during the war of 1914-18? Discuss their legality under the rules of International Law

 

Wednesday, June 1, 1932. 1½ — 4½.
PUBLIC FINANCE.
(OLD AND NEW REGULATIONS.)

  1. What considerations of equity are involved by the action of a Government which reduces rewards to its employees either (a) as a measure to repair a national deficit, or (b) as an example to other employers?
  2. Consider whether there are any grounds for believing that taxation should be progressive if it is to impose equal sacrifice.
  3. “Twenty-five years ago differential taxation was first levied on unearned as against earned income. The time has now come to differentiate between unearned income from fixed-interest securities and that from fluctuating sources.” Is this proposed differentiation (a) desirable (b) feasible?
  4. In what sense, if any can (a) the Income Tax, and 8b) Local Rates be said to enter into the cost of production of goods?
  5. Discuss the possibility of devising a scheme for the taxation of incremental site values which does not place an unfair burden on existing landowners as a class compared with other property owners.
  6. Examine the view that Family Endowment has a good claim to be made a charge on the Exchequer.
  7. If a Board, having a monopoly of the import and export trade of a country and able to buy and sell at current prices at home and abroad, were instructed to maximize its profits, would the effect be more or less advantageous to the country than the flow of trade in free trade conditions?
  8. Compare the economic effects of devaluating the currency of a country by x% in terms of all foreign currencies, with those of imposing an x% ad valorem tariff on all her imports together with an x% ad valorem bounty on all her exports.
  9. What fiscal policy would you adopt if your sole object were to secure the reduction of foreign import duties affecting British exports?
  10. Compare the proper policy of the central authority with that of local authorities with regard to the expansion or contraction in the volume of their outstanding indebtedness.
  11. On what principles should a municipality owning a tramway system proceed in computing the fares to be charged to its passengers?

 

Thursday, June 2, 1932. 1½ — 4½.
THE ECONOMIC DEVELOPMENT OF THE UNITED STATES.

  1. Down to 1860 the economic relations of Great Britain and the United states were complementary rather than rival: after 1870 they were sharply rival.” Comment.
  2. Give examples of the influence of English thought and practice upon the labour movements of America in the early part of the nineteenth century.
  3. Why did the American Mercantile Marine fall away after the Civil War?
  4. Can it be argued that the North and South went to war over any other issue than slavery?
  5. How far was the Tariff responsible for the growth of Trusts? Test your conclusion by reference to particular Trusts.
  6. Distinguish between the “Old” and the “New” Immigration, in respect of (a) its origin, (b) its contribution to American economic life.
  7. Discuss the effect of the disappearance of free land on the economic and social structure of the United States.
  8. Show the significance of the concept of an economic metropolis in an examination of the distribution and range of American industry and commerce.
  9. Indicate, by a sketch map if possible, the regional specialization of American agriculture.
  10. Compare the experience of the United States under the paper dollar of 1862-79 with the experience of England under the paper pound of 1797-1821.

 

Monday, May 30, 1932. 9 — 12.
MONEY.
(NEW REGULATIONS.)

  1. Do you consider that the effects of Peel’s Bank Act were salutary?
  2. Are there good reasons for holding that the “rate of saving” and “rate of investment” should appear as terms in the equation used to demonstrate the principal factors which govern the value of money?
  3. Explain the working of a forward foreign exchange market. How far can it obviate the inconveniences which arise when there is no fixed par of exchange between currencies?
  4. If it had been decided to stabilize the purchasing power of the national standard of value in terms of an index number of prices, what main categories of goods and/or services would you include in the index number?
  5. What considerations have to be taken into account in analyzing the causes which determine the velocity of circulation of money?
  6. Would you have been a mono-metalist or a bi-metalist in the year 1886? State your reasons.
  7. In what respects, if any, do you conceive that the policy of the Federal Reserve System has been open to criticism in the period from 1926 to the present day?
  8. “The resistance of wage earners to reductions in money wages has been of value in saving us from the worst excesses of deflation.” Examine this contention.
  9. “A series of accidents gave India a more satisfactory standard of value than England had in the period from 1873 to 1923.” Discuss.
  10. Do you agree with the view that public works designed to stimulate employment in a slump “are a mere piece of ritual, achieving nothing which could not equally well be achieved by the banking system acting alone, through a sufficiently great alteration in its terms of lending”?
  11. Would the formation of a large “sterling area” be of advantage or disadvantage to this country, in your opinion, if she were endeavouring to maintain a stable currency divorced from gold?

 

Tuesday, May 31, 1932. 1½ — 4½.
LABOUR.
(NEW REGULATIONS.)

  1. Sketch in outline the course followed by real wages in this country during the second half of the nineteenth century, and give some account of the principal causes which brought the changes about.
  2. “The Dole has kept up wages above their proper level.” What are the arguments in support of this view? Discuss their soundness. What changes in public policy, if any, do you conclusions suggest as desirable?
  3. What principal factors determine the magnitude of the change which a fall in the general level of commodity prices occasions in the proportion of the national income accruing to the owners of fixed-interest bearing investments?
  4. With the object of creating employment, a grant is made by the Exchequer to a local authority for a road-widening scheme. The local authority sets about doing the work as efficiently as possible with the aid of steam navies and other labour-saving machinery. Ought the Exchequer to challenge this action? Argue the case for and against.
  5. “The money now paid in unemployment benefit should be used to provide employment.” Suggest possible schemes and discuss their soundness.
  6. Compare the relative levels of wages in skilled and unskilled occupations in this country to-day and before the War. Discuss the bearing of any changes upon (a) the supply of skilled workers, (b) industrial contentment, (c) labour organization and policy.
  7. What are the principal differences between the English and German provisions for State action in the regulation of wages and the settlement of industrial disputes? Discuss whether England could with advantage follow the German example in these matters more closely.
  8. Discuss the relative merits of systems under which wage-rates vary with (a) the market price of the product, (b) a cost of living index, (c) a wholesale price index (d) the profit of the undertaking.
  9. Trace the history of the “One Big Union” idea and movement in this country since 1800.
  10. If you had to measure the change in the general level of money wages in this country since 1900, what difficulties, theoretical and practical, would you encounter?
  11. What were the principal influences determining the development of factory legislation in this country between 1825 and 1878?
  12. What are the functions of the Public Assistance Committees? Do you consider that any of them should be transferred to other authorities or modified in any way? State your reasons.

 

Wednesday, June 1, 1932. 9 — 12.
INDUSTRY.
(NEW REGULATIONS.)

  1. “The entrepreneur has passed away with the Victorian era. His place has been taken by a combination of financier and salaried manager.” Discuss the truth of this contention.
  2. Discuss the possibility of defining “an industry” so as to make it a useful conception in the analysis of economic structure without doing violence to its popular meaning.
  3. Are there any special circumstances in England to-day that make the rational planning of each industry of more importance than in the past? Illustrate what is included under planning from your knowledge of any one industry.
  4. Explain the nature and causes of any differences that occur in the marketing of (a) producers’ goods, (b) consumers’ goods, and the effect of these differences upon the location of the plants making these two types of goods.
  5. How far can statistical proof be adduced for the contention that some manufacturing industries are subject to the law of increasing returns?
  6. Consider the reasons for the introduction and development of municipal trading during the nineteenth century.
  7. How far is it true that since the days of Malthus science and invention have entirely offset any tendency to diminishing returns in the various branches of agriculture?
  8. What forms of integration and combination seem to you to best calculated to reduce the risks of industrial fluctuations, and why?
  9. Discuss the effect on national and international localization of industry of the modern and the possible future development of (a) the telephone, (b) road transport, (c) commercial aviation.
  10. Are we bound to expect a contraction in British industries manufacturing for export? What statistical sources and methods would you use to estimate the extent of such contraction, and to indicate industries and services to which investment might advantageously be directed or transferred?
  11. Discuss any differences between British and foreign systems of education and training in respect of their effect upon the costs of industrial leadership at home and abroad.

 

Thursday, June 2, 1932. 9 — 12.
THEORY OF STATISTICS (OLD REGULATIONS.)
STATISTICS. (NEW REGULATIONS.)

  1. Give an account of the principal properties of the association table, and tests for or measures of association,. Use Table A to illustrate your remark.
TABLE A.
Nervous Symptoms
Women
engaged in
None or slight Marked Total
Factory work 53 30 83
Clerical work 31 7 38
Total 84 37 121
  1. Explain what is meant by a “frequency distribution” and state how, given the data, you would proceed to compile the distribution, directing attention to common faults of presentation. You can use Table B as an illustration. Comment on the dictum “Think in terms of frequency distributions, not of averages.”
TABLE B.
Salaries per day of 10 hours: workers on the French railways, 1896.
Salary Number of workers per 10000
0.25—0.75 10
0.76—1.25 53
1.26—1.75 46
1.76—2.25 145
2.26—2.75 1010
2.76—3.50 3545
3.51—4.25 1921
4.26—5.50 2009
5.51—6.50 702
6.51—7.50 353
7.51—9.00 150
9.10—11.00 37
11.10—15.00 15
15.10—20.00 4
Total 10000

Find some form of average and measure of dispersion for Table B, giving reasons for your choice.

  1. Prove the formula

{\sigma ^2} = {s^2} + {d^2}
where σ is the standard deviation, is the root-mean-square deviation when deviations are measured from an arbitrary origin, and is the difference of the mean from the arbitrary origin.
Find the standard deviation of the data in Table C, and check your own work by any method you consider most effective.

TABLE C.
Deaths from Glanders, persons England and Wales.
1882 3 1892 5
1883 2 1893 6
1884 2 1894 2
1885 5 1895 3
1886 5 1896 1
1887 6 1897 6
1888 2 1898 4
1889 8 1899 5
1890 3 1900 2
1891 4 1901 4
  1. Table D shows the correlation between the class obtained in Part I and the class obtained in Part II by 500 candidates who sat for both parts of the Natural Sciences Tripos in 1922-30. Find the correlation, and the regression equation for class in Part II on class in Part I, and check the fit of the line of regression. The following are the means and standard deviations: suffix 1 refers to Part I and 2 to Part II.
M1= 1.66 σ12 = 0.4337 σ1 = 0.659
M2= 1.93 σ22 = 0.5445 σ2 = 0.738.

 

TABLE D.
Class in Part II
Class in Part II 1 2 3 Total
1 121 35 156
2 92 117 16 225
3 11 72 36 119
Total 224 224 52 500
  1. Explain what is meant by a “weighted” mean, giving illustrations of its use in connection with index numbers of prices.
    Show that ,
    {M_w} = M + r{\sigma _x}\frac{{{\sigma _w}}}{{\bar w}}
    where Mis the weighted mean of X, the arithmetic mean of X, σthe standard deviation of X, σthe standard deviation of the weights w, {{\bar w}} the mean of the weights and the correlation between X and w. In the light of this formula comment on the statement “Weighting usually has little effect.”
  2. What is meant by the “standard error” of a statistical constant? State carefully the conditions assumed, and explain how these limit the value of the standard error as a general measure of “trustworthiness.”
    Prove the formula for the standard error of the arithmetic mean, and find the standard error of the difference between the means M1, Mbelow, σand σbeing the respective standard deviations and N1, Nthe numbers of observations.
M1= 41.3 σ1= 3.8 N1= 100
M2= 38.9 σ2= 3.2 N2= 64.

 

  1. (1) Test the significance of the association in Table A.
    (2) Test whether the data of Table C show anything but mere fluctuations of sampling.
  2. Table E shows the numbers of married women in England and Wales at ages between 15 and 45, reduced to a total of 1000. By some method of interpolation break up the two final decennial groups into quinquennial groups.
TABLE E
Age Married Women
per 1000
15— 7
20— 100
25— 431
35—45 462
1000
  1. Table F shows the beginning of a life-table for Females (no. 7, 1901-10). Explain the meaning of the columns and calculate the figures that should be inserted in the spaces numbered (1), (2) etc. to (9).
    Find also the life-table death-rate and explain why this greatly exceeds the mean crude death-rate for Females 1901-10, viz. 14.4. What light does this phenomenon throw on the probable future course of the crude death-rate?
TABLE F
Age Age
x lx dx px qx Lx Tx x X
0 1000 1174 .8826 .1174 9163 523820 (7) 0
1 8826 (1) (2) (3) (4) (5) (8) 1
2 8494 (6) (9) 2

 

THURSDAY, JUNE 2, 1932. 1½ — 4½.
PRINCIPLES OF POPULATION.
(NEW REGULATIONS.)

  1. Discuss the main features of Hegel’s conception of “economic” or “bourgeois” Society (die bürgerliche Gesellschaft). How far can that conception be regarded as the basis of Marxian theory?
  2. “The expansion of England…was an expansion of Society, and not of the State” (Unwin). Examine the part played by social co-operation, as distinct from political organization, in the development of modern England.
  3. What is the value, and what are the dangers, of the application of biological metaphors (e.g. “the body politic” or “the social organism”) to the study of political theory?
  4. “A true conception of personality and its claims is the first necessity of political theory.” Discuss this statement.
  5. “Liberty is rightly preferred to equality, when the two are in conflict.” Would you agree with this proposition?
  6. What, in your view, is the final basis of the authority of law?
  7. How far does the idea of a “social contract” afford a satisfactory answer to the problem of political obligation?
  8. Does the conception of “the sovereignty of the national State” necessarily involve the unlimited supremacy of such a State?
  9. How far is the conception of “natural rights” a necessary condition of legal progress?
  10. Is the preservation of a distinction between the different “functions” or “powers” of government essential to the liberty of the subject?
  11. What are the political principles implied in the development of “social services” in England during the present century? What limits would you assign on grounds of principle, to the further extension of such services?
  12. “Democracy is not a particular form of State, but the necessary mode of action of all forms of State.” How far would you accept, or on what grounds would you criticize, this proposition?

 

Source:  Cambridge University. Economics Tripos Papers 1931-1933. Cambridge, UK: University Press, 1933, pp. 28-52.

Image Source: Cambridge University, St. John’s Library from website Vintage Postcards.

Categories
Berkeley Chicago Economists Germany Harvard New School Princeton

Harvard. Curriculum vitae submitted by Albert O. Hirschman, ca. 1942

 

One of those serendipitous finds in rummaging through a department’s correspondence in search of one thing (curricular material in my case) is the artifact transcribed for this post, a c.v. submitted to the Harvard department of economics by a 27 or 28 year old Rockefeller Foundation fellow,  O. Albert Hirschmann. It is written in a narrative, autobiographical style as was the custom in Europe of the time. Because I had the great pleasure of having worked as Albert O. Hirschman’s assistant at the Institute for Advanced Study in Princeton during the 1980-81 academic year, I photographed his early c.v. in an act of filial piety. Of course all this and more can be found in the prize-winning biography written by Jeremy Adelman: Worldly Philosopher: The Odyssey of Albert O. HirschmanPrinceton University Press, 2013. Nonetheless, the c.v. possesses the charm of being the original words chosen by Hirschman to market himself back when he was just one of dozens of European economist émigrés looking for steady work.

Thanks to Adelman’s book I learned (p. 203) that one of my Yale mentors, William Fellner, taught a general seminar on the principles of economics at Berkeley that Albert Hirschman took during his Rockefeller Foundation fellowship. Historically speaking, it’s a small world! 

__________________

O. Albert Hirschmann
1751 Highland Place
Berkeley, Calif.

CURRICULUM VITAE

I was born on April 7th, 1915, in Berlin. My nationality is Lithuanian. In 1932 I began to study law and economics at the University of Berlin. In April, 1933, I left for Paris, where I registered at the École des Hautes Études Commerciales (H.E.C.) and at the Institut de Statistiques de l’Université de Paris at the Sorbonne. In 1935 I had obtained the diplomas of both these institutions.

At the end of 1935, I went to England, in order to study for several months at the London School of Economics and Political Science under a scholarship granted to me by the International Student Service, which had already granted to me by the International Student Service, which had already helped me during my former studies. I had courses with Professors Robbins [1898-1984], T. E. Gregory [1890-1970] and B. A. Whale [Philip Barrett Whale, 1898-1950]. I worked in particular under Mr. Whale on French monetary policy since the stabilization of the Franc.

At the end of 1936, after a short stay at Paris, I applied for, and obtained a place as an assistant at the Institute of Statistics of the University of Trieste. I remained there until the middle of 1938, when I was compelled to return to Paris because of the anti-foreign and anti-semitic policy of the Fascist government. At Trieste, I worked under Professor P. Luzzatto-Fegiz [1900-1989]. I became much interested in Population Statistics and a part of my researches in this field was published in an article in the Giornale degli Economisti, January, 1938: “Nota su due recenti tavole di nuzialità della popolazione italiana.” (“A note on two recent nuptiality tables of the Italian population”.) I worked also on several problems of economic statistics and in particular on the statistics of the national income and of family budgets. At the same time I studied for my Doctor’s degree, which I obtained with the grade 120 points in a total of 120, in June, 1938. My thesis was a continuation and an expansion of the work on French monetary policy which I had begun at the London School of Economics. The thesis was to be printed in the Annals of the University, but this was rendered impossible by the subsequent political developments.

While still in Italy, during the first months of 1938, I tried to acquaint myself thoroughly with the Italian financial and economic situation. I finally sent an extensive report to Paris, which was published as a separate booklet, without naming the author, in June, 1938, by the Bulletin Quotidien de la Société d’Études et d’Informations Économiques, under the title: “Les Finances et l’Économie Italiennes – Situation actuelle et perspectives.” This report attracted some attention in Paris because by combining data from various sources I had thrown some light on the Italian economic and financial development which was surrounded by official secrecy. It was upon this report that Professor Charles Rist [1874-1955] offered me to collaborate in his Institut de Recherches Économiques et Sociales. Italy was my special field and from July, 1938, to April, 1940, I wrote regularly three-monthly reports on Italian economic development in L’Activité Économique, which was the publication of the Institute.

I also wrote a small booklet for the above named Bulletin Quotidian on the subject: “L’Industrie Textile Italienne et l’Autarcie.”

In November, 1938, Professor J. B. Condliffe [1891-1981], who was then acting as the director of studies for the International Studies Conference at Bergen, and in this capacity was organizing an international inquiry into the national systems of exchange control, entrusted me with the preparation of a report on the exchange control system of Italy. I also worked on other problems in connection with the Conference and, in particular, devised a new method of measuring the tendency toward bilateralism as completely distinct from the tendency towards equilibrium of foreign trade. Professor Condliffe encouraged me to write a small paper on this idea, and thus I presented two reports at the international Studies Conference at Bergen in 1939: (1) “Le Contrôle des Changes en Italie”—a report of ninety mimeographed pages by the International Institute of Intellectual Cooperation, which for various reasons was not signed, (2) “Étude Statistique de la Tendance du Commerce International [extérieur] Vers l’Équilibre et le Bilatéralisme”—a shorter paper also mimeographed and signed. A recent publication of the U.S. Tariff Commission on “Italian Commercial Policy (1922 – 1940)” has made an extensive use of my report on Italian Exchange Control, whereas Professor Condliffe has quoted my figures on bilateralism in his book “The Reconstruction of World Trade”.

I had registered as a volunteer for the French Army in case of war, in April, 1939. I was called as early as August, 1939. The stationary character of the war gave me the opportunity to prepare still two reports on the Italian economy, the necessary source-material being sent from Paris. After the armistice, in July, 1940, I was demobilized at Nîmes, in Southern France. From there I went to Marseilles, where I met Mr. Varian Fry [1907-1967], who had been sent to Marseilles by the Emergency Rescue Committee in order to evacuate political and intellectual refugees from France. I collaborated with him from August to December, 1940, when, upon the recommendation of Professor Condliffe, I obtained a Rockefeller fellowship, and thereupon the American visa. I arrived in this country on January 14, 1941.

After a short stay in the East, I went to the University of California at Berkeley to work in connection with a research project on Foreign Trade, directed by Professor Condliffe. Soon after my arrival at Berkeley, I met my wife and we were married in June 1941.

My original research plan was to give a statistical analysis of recent quantitative trends in world trade and my first months were spent in working out the specific problems which I intended to study. I wrote several papers on the measurement of concentration and related subjects in descriptive statistics which I hope to publish either as appendices to my main manuscript or as separate journal articles. The next step in my research was to apply the statistical methods which I had worked out to the foreign trade statistics. This required extensive calculations for which Professor Condliffe put an assistant at my disposal. I also participated in several graduate seminars and took a course in the theory of probability.

Upon the renewal of the Rockefeller fellowship for another year and after a two months illness during the winter of 1941-1942, I began to work at the theoretical and historical aspects of the problems which I had first studied from a purely quantitative point of view. The result of my research has now been embodied in a manuscript of 300 pages entitled “National Power and the Structure of Foreign Trade”, of which only the concluding section remains to be written.

Professors Howard S. Ellis [1898-1992] and Condliffe have given me the assurance that the manuscript would be published by a series edited by the newly established Bureau of Economic and Business Research of the University of California. One chapter of the manuscript giving a new statistical analysis of the composition of world trade according to commodity groups, is somewhat loosely connected with the rest and it has been suggested to me to have it published as a separate article. The Rockefeller Foundation has granted me the expenses for a trip to the Middle West and East on which I have just had the opportunity to discuss my manuscript with Professor Viner [1892-1970] at Chicago, Professors Haberler [1900-1995] and Staley [Eugene Alvah Staley (1906-1989) was at Fletcher School of Law and Diplomacy] at Harvard, Professors Staudinger [1889-1980] and Lowe [1893-1995] at the New School of Social Research and with Professor Loveday [1888-1962] and Mr. [Folke] Hilgerdt [1894-1956] of the Economic Intelligence Service of the League at Princeton.

As a result of my training, I have acquired a certain specialization in statistical methods on the one hand and in the field of international economics on the other (theory and history of international trade, international monetary problems, exchange control, foreign trade statistics, etc.) Through my work in Europe I am well acquainted, in particular, with the economic problems of Italy and France.

Having studied for prolonged periods in Germany, France and Italy, I speak and write with complete fluency the languages of these countries. I also have a reading knowledge of Spanish.

 

Source:  Harvard University Archives. Department of Economics, Correspondence & Papers 1902-1950. Box 5, Folder “H”.

Image Source: Albert O. Hirschman before he was dispatched to North Africa, circa 1943. From Michele Alacevich’s Introduction to “Albert Hirschman and the Social Sciences: A Memorial Round-Table” posted July 25, 2015.

Categories
Economists Michigan Suggested Reading Syllabus

Michigan. National Income Syllabus. Gardner Ackley, 1958

 

The following syllabus for Gardner Ackley’s 1958 course on Keynesian macroeconomics was found in the Martin Bronfenbrenner Papers at the Economists’ Papers Archive at Duke University. I have added three short biographical items for this midwestern economist who served as chairman of President Johnson’s Council of Economic Advisers from 1964-68. 

__________________

GARDNER ACKLEY
Minute of the College of Literature, Science, and the Arts

On February 12, 1998, University Professor Emeritus Gardner Ackley passed away at the age of 82, and the economics profession lost one of its true stars.

Gardner Ackley.

  • Served as a distinguished member of the Michigan faculty from 1940 until his retirement in 1984,
  • Chaired the department with great distinction from 1954 to 1961,
  • Published the textbook, Macroeconomics Theory (1961), which defined that field for a generation of economics students around the world,
  • Served President Lyndon Johnson as Chairman of his Council of Economic Advisers from 1964 to 1968,
  • Served the nation as American Ambassador to Italy during 1968-69, and,
  • Was rewarded by his profession with election to the Presidency of the American Economic Association in 1982.

These accomplishments and honors distinguish the career of Gardner Ackley as among the most stellar of his generation, and they define a standard worthy to inspire the succeeding generations of professional economists.

No celebration of Gardner Ackley, however, should conclude without mentioning at least the following two of the many significant challenges he shouldered during his distinguished career. His years as Chair of the Michigan Department of Economics included the period of challenge to academic freedom and McCarthyism. Gardner stood solidly in support of students and colleagues in those years in ways that marked him as a man of extraordinary courage and integrity. Later, as Chair of the Council of Economic Advisers, he stood up once again for what he believed, this time challenging the President of the United States to adopt a proper, if unpopular, anti-inflationary policy as the war in Vietnam stretched the economic capacities of an already fully-employed nation.

Those of us who knew Gardner, whether as a colleague, mentor, or teacher, feel privileged to have known him and appreciate the dimension of his loss.

Source:  University of Michigan, Faculty History Project. LSA Minutes, Gardner Ackley.

__________________

From the University of Michigan Alumni Magazine
February 13, 1954

GARDNER ACKLEY, AM. ’37, Ph.D. ’40, who was named Chairman of the Department of Economics effective February 1, has divided his time almost equally since 1940 between the University and the Federal government. He joined the faculty in 1940 as an Instructor, and became a full Professor in 1952. Professor Ackley’s government service has been with the National Resources Planning Board, the wartime OPA, the Office of Strategic Services, the Economic Stabilization Agency, and, during 1951 and 1952, as Economic Advisor and Assistant Director of the Office of Price Stabilization. Professor Ackley earned his A.B. at Western Michigan College; he is a member of the American Economic Association and the Econometric Society. He has served on numerous University committees, including the Board in Control of Intercollegiate Athletics.

Source: Classroom Profile. The Michigan Alumnus (February 13, 1954) p. 214.

__________________

Biography from Guide to Gardner Ackley Papers

(Hugh) Gardner Ackley was born in Indianapolis, Indiana on June 30, 1915. In 1936, Ackley received his baccalaureate degree from Western State Teachers College (now Western Michigan University) in Kalamazoo, Michigan. In 1937, a master’s degree was conferred upon Ackley by the University of Michigan and in 1940 he received a doctoral degree from the same institution.

In 1939 and 1940, Ackley was an instructor at Ohio State University, returning to teach at the University of Michigan in late 1940. Throughout the Second World War he served as a member of the government in Washington. From 1941 to 1942, and again from 1944 to 1946 he worked at the Office of Price Administration. In 1943 and 1944 he was assigned to the Office of Strategic Services. Concluding his wartime service, Mr. Ackley returned to the University of Michigan where he resumed his academic career in the Department of Economics.

Ackley returned to government service in 1951, serving for two years as the assistant director of the Office of Price Stabilization. After completing this assignment he once again returned to Ann Arbor to carry on scholarly pursuits.

Gardner Ackley left Ann Arbor for Washington for the third time in August 1962 when President John Kennedy named him as a member of the Council of Economic Advisors. He served as a member of the council until November 14, 1964, when President Lyndon Johnson named him the new chairman of the CEA. Early in 1968, Mr. Ackley left the council to become ambassador to Italy, a post he held until 1969. After returning from Italy, Ackley again resumed his academic career at the University of Michigan.

From 1969 until 1984 Ackley was the Henry Carter Adams University Professor of Political Economy at the University of Michigan. During this same period, he served on many national commissions and councils devoted to economic issues. He was a member of the Trilateral Commission from 1977 to 1983 and during 1978-1979 he was a member of the Advisory Council on Social Security. He also served as president of the American Economics Association during 1982.

Among his many honors, Ackley received the Distinguished Faculty Achievement Award at the University of Michigan in 1976 and he was also elected a fellow of the American Academy of Arts and Science. Ackley has been professor emeritus since 1984.

Gardner Ackley died February 12, 1998.

Source:  University of Michigan. Bentley Historical Library. Guide to Papers of Gardner Ackley.

__________________

Syllabus 1958

[Handwritten note by Bronfenbrenner: G. Ackley (Michigan)]

Economics 151
NATIONAL INCOME I
Reading List

Second Semester, 1957-58

In the following list selected readings on topics to be covered in the course are arranged under two headings: “Assignments,” which all students should study, and “References,” which usually (but not always) present a more advanced or more specialized treatment, or a conflicting point of view. No effort is made to supply references on topics only lightly touched on in the course. Publisher and date of publication are given only upon first listing. Additional assignments may be made in class.

All students should purchase J. M. Keynes, General Theory of Employment, Interest, and Money, Harcourt-Brace, 1936. (This will be referred to as “Keynes.”) In addition, each student should purchase, at the Cashier’s Office in the Administration Building, a “lab ticket” for this course (price $3.75). This will entitle him to receive a copy of the preliminary edition of G. Ackley, An Introduction to Macroeconomic Theory, which will be distributed in class. (This will be referred to as “Ackley.”)

Modern textbooks which treat the general field covered by this course include the following:

  1. R. and N. Ruggles, National Income and Income Analysis (McGraw-Hill, 2nded., 1956).
  2. T. Schelling, National Income Behavior: An Introduction to Algebraic Analysis (McGraw-Hill, 1951)
  3. A. P. Lerner, Economics of Employment (McGraw-Hill, 1951)
  4. S. Weintraub, Income and Employment Analysis (Pitman, 1951)
  5. R. V. Clemence, Income Analysis (Addison-Wesley, 1951)
  6. J. P. McKenna, Aggregate Economic Analysis (Dryden, 1955)
  7. T. Morgan, Income and Employment (Prentice-Hall, 2nded., 1952)

In this reading list no references are given to these textbooks. The student who wishes to use any of them can, however, easily find the appropriate sections from the table of contents.

Two other books which many students find helpful in understanding Keynes’ General Theory are A. H. Hansen, Guide to Keynes (McGraw-Hill, 1953); and D. Dillard, The Economics of J. M. Keynes (Prentice-Hall, 1948).

There will be one or more written problems which all students will be expected to hand in. Graduate students will be expected to write a paper.

  1. INTRODUCTION (Feb. 6,8)

Assignments:

  1. Ackley, Ch. I.

References:

  1. K. Kurihara, Introduction to Keynesian Dynamics (Columbia University Press, 1956), ch. 1

*  *  *  *  *  *

  1. MEANING AND MEASUREMENT OF NATIONAL INCOME AND PRODUCT  (Feb. 11 – 18)

Assignments:

  1. Ackley, Chs. II and III.
  2. National Income Supplement to the Survey of Current Business, 1954 edition
    (U.S. Dept. of Commerce, 1954), Parts I and II. Every student is urged to purchase this useful volume.)

References:

  1. National Income Supplement, Part III.
  2. A System of National Accounts and Supporting Tables, Studies in Methods, No. 2 (Series F). (United Nations, 1953)
  3. A Simplified System of National Accounts (Organization for European Economic Cooperation, 1952)
  4. H. C. Edey and A.T. Peacock, National Income and Social Accounting, (Hutchinson’s University Library, 1954)
  5. R. and N. Ruggles, National Income Accounts and Income Analysis. (McGraw-Hill, 1956) pp. 3-210.
  6. C. S. Shoup, Principles of National Income Analysis. (Houghton, Mifflin, 1947
  7. S. Kuznets, National Income: A Summary of Findings. (National Bureau of Economic Research, 1946) Especially pp. 111-139.
  8. C. S. Shoup, “Development and Use of National Income Data,” in Survey of Contemporary Economics. (H.S. Ellis, ed., Blakiston, 1949), pp. 288-313.
  9. S. Kuznets, “National Income and Economic Welfare,” in Economic Change. (Norton, 1953), pp. 145-215.
  10. J. R. Hicks, “Valuation of Social Income,” Economica, Vol. VII (new series), May 1940
  11. K. E. Boulding, “Income or Welfare,” Review of Economic Studies, Vol. XVII, 1949-50.
  12. J. P. Powelson, Economic Accounting. (McGraw-Hill, 1955), Chs. 14-20.
  13. Keynes, Ch. 6.
  14. The National Economic Accounts of the United States: Hearings before the Subcommittee on Economic Statistics of the Joint Economic Committee (U.S. Govt. Printing Office, 1957), esp. pp. 101-299.

*  *  *  *  *  *

  1. OUTPUT AND EMPLOYMENT (Feb. 20 – 25)

Assignments:

  1. National Income Supplement, Part IV.
  2. G. Bancroft, “Current Unemployment Statistics of the Census Bureau and Some Alternatives,” in The Measurement and Behavior of Unemployment (Princeton University Press, 1957), pp. 63-119.

References:

  1. Keynes, Ch. 4
  2. M. Gilbert and I.B. Kravis, An International Comparison of National Products and the Purchasing Power of Currencies (Organization for European Economic Cooperation, 1954). Also see references under II, especially items 7 and 9.
  3. Other papers in Measurement and Behavior of Unemployment.

*  *  *  *  *  *

  1. THE “CLASSICAL” ECONOMICS (Feb. 27 – March 8)

Assignments:

  1. Ackley, Chs. V, VI.
  2. Keynes, Ch. 2
  3. A. H. Hansen, Monetary Theory and Fiscal Policy (McGraw-Hill, 1949) Ch. 3
  4. Ackley, Chs. VII, VIII.

References:

  1. F.M. Taylor, Principles of Economics(Ronald, 9thed., 1921), pp. 196-205.
  2. K. Wicksell, Lectures on Political Economy(English translation by L. Robbins, Routledge and Kegan Paul, 1935), Vol. II, especially pp. 159-208.

*  *  *  *  *  *

  1. OBSTACLES TO FULL EMPLOYMENT (March 11 – 15)

Assignments:

  1. Ackley, Ch. IX
  2. Keynes, Chs. 13, 15.

References:

  1. A. H. Hansen, Monetary Theory and Fiscal Policy, Ch. 4.
  2. A. P. Lerner, “Interest Theory,” in The New Economics (S.E. Harris, ed., Knopf, 1947), pp. 655-661.
  3. L. R. Klein, The Keynesian Revolution (Macmillan, 1949), pp. 69 (top), – 73 (Middle), 117-123.
  4. Readings in the Theory of Income Distribution. (Selected by a Committee of the American Economic Association, Blakiston, 1946), articles by Keynes, Robertson, Hicks, Somers, and Lutz.
  5. A. G. Hart, Money, Debt, and Economic Activity. (Prentice-Hall, 1948), Ch. 8.
  6. Keynes, General Theory, Ch. 17.
  7. D. H. Robertson, “Some Notes on the Theory of Interest,” in Money, Trade, and Economic Growth (Macmillan, 1931), pp. 193-209.
  8. S. C. Tsiang, “Liquidity Preference and Loanable Funds Theories, Multiplier and Velocity Analyses: A Synthesis,” American Economic Review, XLVI (Sept. 1956), pp. 539-555.
  9. G. Ackley, “Liquidity Preference and Loanable Funds Theories of Interest: Comment,” ibid., XLVII (Sept. 1957), pp. 662-73.
  10. K. Kurihara, Introduction to Keynesian Dynamics, ch. 4.

*  *  *  *  *  *

  1. HOUR EXAMINATION (March 18)

*  *  *  *  *  *

  1. CONSUMER EXPENDITURES AND THE SIMPLE KEYNESIAN MODEL (March 20 – April 2)

Assignments:

  1. Ackley, Ch. X.
  2. Keynes, Chs. 1, 3, 5, 8, 9.
  3. G. Katona, “The Variability of Consumer Behavior and the Survey Method,” in Contributions of Survey Methods to Economics (L.R. Klein, ed., Columbia Univ. Press, 1945) pp. 49-67, 78-88.

References:

  1. R. Ferber, A Study of Aggregate Consumption Functions. (Technical Paper 8, National Bureau of Economic Research, 1953)
  2. Ruth Mack, “Economics of Consumption,” in A Survey of Contemporary Economics, Vol. II, with comment by J.S. Davis and J. Marschak (ed. B. Haley, Irwin, 1952), Ch. 2.
  3. G. Katona, Psychological Analysis of Economic Behavior. (McGraw-Hill, 1951), pp. 63-192.
  4. G. Katona and E. Mueller, Consumer Attitudes and Demand, 1950-52. (Survey Research Center, 1956).
  5. G. Katona and E. Mueller, Consumer Expectations, 1953-1956. (Survey Research Center, 1956).
  6. Savings in the Modern Economy, ed. W.W. Heller, F.M. Boddy, and C.L. Nelson. (Univ. of Minnesota Press, 1953), Chs. 7, 8, 13.
  7. E. F. Denison, “Saving in the National Economy,”Survey of Current Business, January 1955, pp. 8-24.
  8. I. Friend (with V. Natrella), Individual Saving: Volume and Composition (John Wiley, 1954). Especially pp. 118-154.
  9. R. Klein, ed., Contributions of Survey Methods to Economics.
  10. J. S. Duesenberry, Income, Saving, and the Theory of Consumer Behavior, (Harvard University Press, 1949)
  11. J. S. Duesenberry, “Income-Consumption Relations and their Implications,” in Income, Employment, and Public Policy. (Norton, 1948), pp. 54-81.
  12. J. Tobin, “Relative Income, Absolute Income, and Saving,” in Money, Trade, and Economic Growth. (Macmillan, 1951), pp. 135-156.
  13. J. Tobin, “Asset Holdings and Spending Decisions,” American Economic Review, XLII (May 1952) pp. 109-123.
  14. J. R. Hicks, A Contribution to the Theory of the Trade Cycle (Oxford University Press, 1950), Chs. 2, 3.
  15. R. F. Harrod, Towards a Dynamic Economics, (Macmillan, 1949), pp. 35-62.
  16. A. C. Pigou, Employment and Equilibrium (2ndedition, Macmillan, 1949), pp. 28-46.
  17. M. Friedman, A Theory of the Consumption Function (Princeton University Press, 1957)
  18. K. Kurihara, Introduction to Keynesian Dynamics, ch. 2.
  19. F. Modigliani and R. Brumberg, “Utility Analysis and the Consumption Function: An Interpretation of Cross-Section Data” in K. Kurihara (ed.), Post-Keynesian Economics. (Rutgers University Press, 1954), pp. 388-436.

*  *  *  *  *  *

  1. APPLICATIONS AND EXTENSIONS OF THE SIMPLE KEYNESIAN MODEL (Apr. 3, 5, 15, 17)

Assignments:

  1. Ackley, Ch. XII.
  2. Keynes, Chs. 7, 10.
  3. P. A. Samuelson, “Simple Mathematics of Income Determination,” in Income, Employment, and Public Policy, pp. 133-155.

References:

  1. G. Haberler, “Mr. Keynes’ Theory of the Multiplier,” in Readings in Business Cycle Theory, selected by a committee of the American Economic Association (Blakiston, 1944), pp. 193-202.
  2. F. Machlup, “Period Analysis and Multiplier Theory,” in Readings in Business Cycle Theory, pp. 203-234.
  3. R. M. Goodwin, “The Multiplier,” in The New Economics. (S.E. Harris, ed., Knopf, 1947), pp. 482-499.
  4. G. Ackley, “The Multiplier Time Period,” American Economic Review, June 1951.
  5. R. Turvey, “Some Notes on Multiplier Theory,” American Econ. Review, June 1953
  6. L. A. Metzler, “Three Lags in the Circuit Flow of Income,” in Income, Employment and Public Policy, pp. 11-32.
  7. A. P. Lerner, “Saving Equals Investment,” in The New Economics, pp. 619-626.
  8. L. R. Klein, The Keynesian Revolution, pp. 75 (bottom) – 80 (top), 110-117.
  9. F. A. Lutz, “The Outcome of the Saving-Investment Discussion,” in Readings in Business Cycle Theory, pp. 158-168.
  10. A. P. Lerner, “Saving and Investment,” in Readings in Business Cycle Theory, pp. 158-168. [sic]
  11. A. H. Hansen, Monetary Theory and Fiscal Policy (McGraw-Hill, 1949), pp. 219-225.
  12. A. H. Hansen, Business Cycles and National Income (Norton, 1951), pp. 156-163, 606-616.
  13. K. Kurihara, Introduction to Keynesian System, chs. 5, 6, 7.

*  *  *  *  *  *

  1. THE COMPLETE KEYNESIAN SYSTEM (April 19 – May 1)

Assignments:

  1. Ackley, Chs. XIII, XIV
  2. Keynes, Chs. 14 (Appendix optional), 18, 19 (Appendix optional), 20 (Sec. I optional) 21 (Sec IV optional), 24.
  3. A. P. Lerner, The Economics of Control. (Macmillan, 1944), Chs. 22, 23.

References:

  1. A. H. Hansen, Monetary Theory and Fiscal Policy, Ch. 5, 7, 8, 9.
  2. J. R. Hicks, “Mr. Keynes and the ‘Classics’,” in Readings in the Theory of Income Distribution, pp. 461-476.
  3. O. Lange, “The Rate of Interest and the Optimum Propensity to Consume,” in Readings in Business Cycle Theory, pp. 169-178 (only).
  4. D. Patinkin, “Price Flexibility and Full Employment,” in Readings in Monetary Theory. (eds. F.A. Lutz and L.W. Mintz, Blakiston, 1951), pp. 252-58.
  5. F. Modigliani, “Liquidity Preference and the Theory of Interest and Money,” Readings in Monetary Theory, pp. 186-239.
  6. A. P. Lerner, “Relation of Wage Policies and Price Policies,” in Readings in the Theory of Income Distribution, pp. 314-329.
  7. L. G. Reynolds, “Relations between Wage Rates, Costs, and Prices,” in Readings in the Theory of Income Distribution, pp. 294-313.
  8. J. Tobin, “Money Wage Rates and Employment,” in The New Economics, pp. 572-587.
  9. J. Lintner, “The Theory of Money and Prices,” in The New Economics, pp. 503-537.
  10. L. R. Klein, The Keynesian Revolution, pp. 72-75, 80-90, 106-110, 199-206.
  11. L. R. Klein, “Theories of Effective Demand and Employment,” Journal of Political Economy, April, 1947.
  12. E. S. Mason, “Prices, Costs, and Profits,” in Money, Trade, and Economic Growth, pp. 177-190.
  13. J. R. Schlesinger, “After Twenty Years: The General Theory,” Quarterly Journal of Economics (November 1956), pp. 581-602.
  14. “Keynesian Economics after Twenty Years,” (Papers and comments by W. Fellner, D. Dillard, D. Wright, W.A. Salent, and T. Scitovsky), American Economic Review, XLVII (May 1957), pp. 67-95.
  15. A.C. Pigou, Keynes’ General Theory(Macmillan, 1950).
  16.  K. Kurihara, Introduction to Keynesian Dynamics, ch. 10.

*  *  *  *  *  *

  1. HOUR EXAMINATION (May 3)

*  *  *  *  *  *

  1. INFLATION (May 6, 8)

Assignments:

  1. A. H. Hansen, Monetary Theory and Fiscal Policy, ch. 11
  2. A. P. Lerner, Economics of Employment, chs. 13, 14 (15 and 16 optional).

References:

  1. K. Kurihara, Introduction to Keynesian Dynamics, ch. 8.
  2. A. Smithies, “The Behavior of Money National Income under Inflationary Conditions,” Quarterly Journal of Economics, LVII (1942)
  3. T. Koopmans, “The Dynamics of Inflation,” Review of Economic Statistics, XXIV (1942)
  4. J. Duesenberry, “The Mechanics of Inflation,” and F. Holzman, “Income Determination in Open Inflation,” Review of Economics and Statistics, XXXII (1950).
  5. B. Hansen, A Study in the Theory of Inflation (Allen and Unwin, 1951), especially ch. VII.

*  *  *  *  *  *

  1. THE THEORY OF INVESTMENT (May 10 – 17)

Assignments:

  1. Ackley, Ch. XV.
  2. Keynes, Chs. 11, 12, 16, 22.
  3. J. R. Meyer and E. Kuh, The Investment Decision (Harvard Univ. Press, 1957), Ch. 1.

References:

  1. A. P. Lerner, Economics of Control, Ch. 25
  2. L. R. Klein, Keynesian Revolution, pp. 62-69.
  3. A. Smithies, “Economic Fluctuations and Growth,”Econometrica, 25 (January 1957), pp. 1-52.
  4. K. Kurihara, Introduction to Keynesian Dynamics, ch. 3.

*  *  *  *  *  *

  1. ECONOMIC POLICY AND THE PRESENT SITUATION (May 20 – 27)

Assignments:

  1. Lerner, Economics of Control, Ch. 24.
  2. M. Friedman, “A Monetary and Fiscal Framework for Economic Stability,” in Readings in Monetary Theory, pp. 369-393.
  3. To be announced.

*  *  *  *  *  *

    1. MISCELLANEOUS FURTHER REFERENCES:

  1. J. Robinson, Essays in the Theory of Employment (Macmillan, 1937).
  2. R. F. Harrod, The Life of John Maynard Keynes. (Harcourt-Brace, 1951).
  3. J. A. Schumpeter, “John Maynard Keynes, 1883-1946,” American Economic Review, Sept. 1946 (reprinted in The New Economics, pp. 73-101).
  4. The New Economics, Chs. 1-8, 11-19, 31-35, 39, 41, 42.
  5. A. F. Burns, Economic Research and the Keynesian Thinking of our Times (26thAnnual Report of the National Bureau of Economic Research, Inc., June 1946).

 

Source:  Duke University. David M. Rubenstein Rare Book & Manuscript Library. Economists’ Papers Archive. Martin Bronfenbrenner Papers, Box 25, Folder “Macro-Econ, n.d.”.

Image Source:  University of Michigan, Faculty History Project, Gardner Ackley page.

 

Categories
Exam Questions Harvard

Harvard. Final Exams for History of Tariff Legislation. Taussig 1883/4-1889/90

 

Frank W. Taussig first taught his half-course “History of U.S. Tariff Legislation” (one hour of class a week for the entire academic year) in 1883-84.  Beginning 1886-87 the half-course met two hours per week during the second term only. The previous post provides the entire 28 page printed syllabus with bibliography for this course dated 1888. Today’s post provides enrollment data for the course from 1883-84 through 1889-90 along with all the end-of-term examinations for the course.

_________________

Announcement of new course on Tariff Legislation

The scheme of instruction for the year 1883-84….Of the remaining five courses, of which four are new, two are full courses, with three or two exercises a week, while three, having each one exercise a week, are rated as half-courses. The latter are devoted to the treatment of special topics: The Economic Effects of Land Tenures in England, Ireland, France, and Germany, by Professor Laughlin; Tariff Legislation in the United States, by Dr. Taussig; Comparison of the Financial Systems of France, England, Germany, and the United States by Professor Dunbar.”

Source: Harvard University. Annual Report of the President of Harvard College 1882-1883, p. 73.

_________________

Course Enrollments

Enrollment 1883-84

Dr. Taussig. 6. Lectures on the History of Tariff Legislation, chiefly in the United States with a discussion of the principles of tariff legislation.  Hours per week: 1.

Total 17:  13 Seniors, 1 Junior, 3 Graduates

Source: Harvard University. Annual Report of the President of Harvard College 1883-1884, p. 72.

*  *  *  *  *

Enrollment 1884-85

Dr. Taussig. 6. History of Tariff Legislation in the United States, with a discussion of principles.—Lectures.  Hours per week: 1. *Consent of instructor required.

Total 40:  26 Seniors, 10 Juniors, 1 Graduate, 3 Other.

Source: Harvard University. Annual Report of the President of Harvard College 1884-1885, p. 86.

*  *  *  *  *

Enrollment 1885-86

Dr. Taussig. 6. History of Tariff Legislation in the United States.— Discussion of principles.  Hours per week: 1. *Consent of instructor required.

Total 35:  17 Seniors, 11 Juniors, 2 Graduates, 5 Others.

Source: Harvard University. Annual Report of the President of Harvard College 1885-1886, p. 51.

*  *  *  *  *

Enrollment 1886-87

Dr. Taussig. 6. History of Tariff Legislation in the United States, and consideration of its economic effects.—Lectures, written exercises, and oral discussion.  Hours per week: 2, 2ndhalf-year. *Consent of instructor required.

Total 38:  28 Seniors, 4 Juniors, 2 Graduates, 4 Others.

Source: Harvard University. Annual Report of the President of Harvard College 1886-1887, p. 59.

*  *  *  *  *

Enrollment 1887-88

Dr. Taussig. 6. History of Tariff Legislation in the United States.—Lectures, required reading, and investigation of special topics.  Hours per week: 2, 2ndhalf-year. *Consent of instructor required.

Total 58:  31 Seniors, 17 Juniors, 5 Graduates, 5 Others.

Source: Harvard University. Annual Report of the President of Harvard College 1887-1888, p. 62.

*  *  *  *  *

Enrollment 1888-89

Dr. Taussig. 6. History of Tariff Legislation in the United States.—Lectures and reports on special topics. Hours per week: 2, 2ndhalf-year. *Consent of instructor required.

Total 34:  18 Seniors, 14 Juniors, 1 Sophomore, 1 Other.

Source: Harvard University. Annual Report of the President of Harvard College 1888-1889, p. 72.

*  *  *  *  *

Enrollment 1889-90

Dr. Taussig. 6. History of Tariff Legislation in the United States.—Lectures on the History of Tariff Legislation.—Discussion of brief theses (two from each student).—Lectures on the Tariff history of France and England.  Hours per week: 2 or 3, 2ndhalf-year. *Consent of instructor required.

Total 29:  19 Seniors, 9 Juniors, 1 Other.

Source:Harvard University. Annual Report of the President of Harvard College 1889-1890, p. 80.

_________________

1883-84.
POLITICAL ECONOMY 6
[Mid-Year]

  1. Give a brief summary of the contents of Hamilton’s Report on Manufactures. Comment on his discussion of the relative productiveness of agriculture and manufactures; and on the proposition that manufactures are peculiarly productive, and particularly desirable in a country, because they admit of a greater division of labor and more extended use of machinery.
    Make some comparison between the general character of Hamilton’s Report and Gallatin’s Memorial, of 1831, on the Tariff.
  2. Describe the tariff act of 1789. Should you consider it a protective measure?
  3. Give a brief history of the cotton manufacture from 1789 to 1824, and of the tariff legislation on cottons. Comment on the following: “It is seen that the manufacture of coarse cotton cloth has been more efficiently and steadily protected than any other and that such cloths are now supplied so cheaply that as to enter largely into the list of exports….The more perfectly the home market is secured to the domestic artisan, the greater is the tendency to cheapening the commodity.”— H.C. Carey.
  4. Give an account of the passage of the tariff act of 1828, and of the provisions of that act. Why was it called “the tariff of abominations”? Comment on this statement: “Next came the tariff of 1828, the first that was based on the idea of protection for the sake of protection.”
  5. What was the “forty-bale theory,” or “export tax theory” of Congressman McDuffie? Discuss, in connection with it, the incidence of taxation by duties on imports.
  6. Given the important provisions of the Compromise Act of 1833. How long was that act, by its terms, to remain in force, and how long did it remain in force? Criticize the tariff system which the act finally brought into operation. Comment briefly on the following: “Mr. Calhoun introduced and carried the scheme of what is called a revenue tariff, which, taking off by gradations, should finally reduce the income, through the custom house, to the measure necessary to support the Government, and adjust it on the principles of a tariff for revenue only. And how long did it take this beneficent measure…to do its work on the industries of this country? In 1837 a bankruptcy covered the whole land, without distinction of sections, with ruin.”—W.M. Evarts.

Mid-Year. 1884.

_________________

1883-84.
POLITICAL ECONOMY 6
[End-Year]

  1. Secretary Walker, in his report on the tariff in 1845, laid down these general rules:—
    No duty should be imposed above the lowest rate that will yield the largest revenue.
    Below such a rate discrimination may be made.
    The maximum revenue duty should be imposed on luxuries.
    Should you say that these rules were sound, and stated the proper principles applicable to import duties? Should you say that the legislation based on them in the tariff of 1846 was a sound application of the principles of free trade?
  2. Describe and discuss the plan on which the wool and woolens schedule of the existing tariff was formed.
  3. Compare the tariff history of France during and after the wars of the French Revolution, with that of the United States during and after the war of the rebellion.
  4. Comment on the following:—
    “A tax on raw materials is not like a tax on finished goods. A tax on raw materials is equal to its own amount, plus the usual percentage of gross profit, multiplied by the number of procedures through which it has to pass until it reaches the consumer in the finished state. A protection of $28,000,000 on raw wool [a duty of 10 cents a pound, with a domestic production of 280,000,000 pounds] keeps swelling and swelling at each intermediate stage till it reaches the consumer, and may be called nearly a hundred million dollars when it reaches the consumer it its most finished state.”
  5. What should you say of the tariff as a factor in the general prosperity of the United States during the past hundred years?

Ann. June, 1884.

_________________

1884-85.
POLITICAL ECONOMY 6
[Mid-Year]

(Omit either question 3 or question 4.)

  1. Comment briefly on the following:—
    “There is not a single great branch of domestic manufactures which had not been established in some form in this country long before a protective tariff had been or could have been imposed. The manufacture of iron is nearly as old as the history of every colony or territory in which there is any iron ore. The manufacture of woolens is as old as the country itself, and was more truly a domestic manufacture when our ancestors were clothed with homespun than now. The manufacture of cotton is almost as old as the production of the fibre on our territory.”
  2. Compare the tariff act of 1816 with that of 1824, noting differences in (1) the general range of duties, (2) the circumstances under which they were passed, (3) the action taken in regard to them by the representatives of New England, the Middle States, and the South. It has been said that “the tariff of 1816 marks the beginning of protection in this country,” and that “the tariff of 1824 was our first tariff worthy of the name of protection.” Which of these statements is true, if either?
  3. Comment on the following:—
    “No protective duty was ever levied on a single article, the home manufacture of which grew to large proportions under that duty, without the price to the consumer growing cheaper, the duty thus being a boon instead of a tax.”
    “A duty on an imported article is invariably added to its price, at the cost of the buyer, and added also to the price of like articles made here.”
  4. State carefully the argument for the protection of young industries and mention the conditions, if any, which might justify the application of such protection.
  5. Give a brief critical statement of the views expressed by Hamilton, Gallatin, Clay, and Webster on the protective controversy.

Mid-year. 1885

_________________

1884-85.
POLITICAL ECONOMY 6
[End-Year]

  1. State as nearly as you can the duties on the following articles from 1846 to 1884: pig-iron, steel-rails, wool, woolen cloths, silks, coffee, copper
    Take any one of the following articles: pig-iron, wool, woolen cloths, silks, copper; and say something as the economic effect of the duties on that one between 1860 and 1884.
  2. Give an account of the tariff act of 1864. Compare the tariff policy adopted in the United States after the close of the civil war, and with the policy of France after 1815.
  3. What has been the practice in our tariff acts since 1842 as regards the imposition of specific and ad valorem duties? Comment on the following: “It is an economic truth that the ad valorem system is the only equitable rule for assessing duties. With the whole power of a great government behind, there is no reason why the laws of the country should not be enforced. The outcry of undervaluation is simply a trick to blind the people, as it would be impossible to enact a law imposing duties of 80, 100, even 200 percent. in the plain unvarnished form of ad valorem duties.”
  4. Comment briefly on two of the following:—
    1. “The fairest and most satisfactory test of the effect of the tariff on prices is to compare prices of the same article under high and low tariffs. The average gold price of pig-iron before 1860 was $28.50 per ton; in recent years it has been $33.70. The average is higher by $5.20 under a high tariff than during the period of low duties.”
    2. “Nothing can be more false than the claim of free trade advocates than that a duty is a tax that comes out of the farmers and artisans of this country. By far the greater part of the revenue collected on importations is the toll paid by people of other countries for the admission of their goods….I was assured by a score of manufacturers in England that the recent increase in the French tariff came out of their pockets, and not from the consumers in France; that they were compelled to sell their goods in France at the same price as before the increase of duty.”
    3. “A conclusive answer to the assertion that the protective policy secures high wages to the laborers of this country, is found in the fact that wages are higher in the United States—absolutely and in comparison with the old world rates—in those industries which do not have, or confessedly do not need, protection.”
  5. Compare the grounds on which a policy of protection has been advocated in recent years with the grounds put forward in 1820-30, and give any reasons that may occur to you for changes in the arguments.

Ann. June. 1885.

_________________

1885-86.
POLITICAL ECONOMY 6
[Mid-Year]

  1. Comment on the following:—
    “Beside the protection thrown over the manufacturing interest by Congress during this period (1789-1812), the war which raged in Europe produced a favorable effect. As the United States was a neutral nation, she fattened on the miseries of the European nations, and her commerce increased with astonishing rapidity. Our manufactures flourished from the same cause, though not to a corresponding degree with our commerce.”
  2. Take two of the following:—
    (a) Give some account of the sources from which we learn the character of the act of 1828, and the circumstances under which it was passed.
    (b) What was Webster’s position on the tariff question, in 1824, in 1828, and in 1833?
    (c) What was Clay’s position on the tariff question in 1820, in 1828, and in 1832?
    Under (b) and (c) discuss briefly the reasons why Clay and Webster acted as they did at the dates mentioned.
  3. Comment on the following:—
    “Whenever we diminish importation by a protective tariff, we must at the same time diminish the production of those goods which, were trade free, we should given in exchange for the goods imported…..It would, however, be a mistake in the other direction to assume that all the industry set in operation by the tariff is withdrawn from other employments, and that there is no increase whatever. The very fact that, under free trade, goods are imported instead of being made at home shows that we find it easier to make the goods which we send abroad than to make those which we receive in exchange for them. Hence when we are forced to make them for ourselves, there must be an increase in the sum total of our industry.”
  4. Comment on the following, and state when and by whom you think it was written:—
    “The principal argument for the superior productiveness of agricultural labor turns on the allegation that the labor employed on manufactures yields nothing equivalent to the rent of the land, or to that net surplus, as it is called, which accrues to the proprietor of the soil….It seems to have been overlooked that the land itself is a stock or capital, advanced or lent by its owner to the occupier or tenant, and that the rent he receives is only the ordinary profit of a certain stock in land, not managed by the proprietor himself, but by another, to whom he lends or lets it, and who, on his part, advances a second capital, to stock and improve the land, upon which he also receives the usual profit. The rent of the landlord and the profit of the farmer are, therefore, nothing more than the ordinary profit of two capitals belonging to two different persons, and united in the cultivation of the farm.”
  5. State as nearly as you can what were the duties on cotton goods, woolen goods, bar iron, hemp, and articles not specifically provided for, in the years 1800, 1814, 1820, 1830, and 1837. Mention what tariff act was in force at each date, and whether the duty was specific or ad valorem. Use tabular form if you wish.

Mid-year. 1886.

_________________

1885-86.
POLITICAL ECONOMY 6
[End-Year]

[Omit one question.]

  1. Does a tax on imports operate as a tax on exports? Apply your reasoning to the exports of Southern cotton in 1830, and to those of Western grain in 1880.
  2. Assuming that you were called on to reduce duties, state the order of preference in which you would effect reductions in the present duties on iron, sugar, silks. Give your reasons.
  3. Make a comparison between the general course of tariff legislation in the United States and on the continent of Europe, from 1860 to the present time.
  4. Make a comparison between the tariff legislation of the United States in 1833 and in 1846.
  5. Comment on the reasoning and the statement of fact in the following:—
    “The duty of 1867 on wool, which gave to wool-growing its greatest encouragement, has added nothing to the cost of wool to the manufacturer or the consumer. On the contrary, the price has been greatly cheapened. In 1867 the price was 51 cents, in 1870 it was 46 cents, in 1875 it was 43 cents. There has been a steady reduction, with occasional fluctuations, since 1867. Free wool will be of no permanent benefit to manufacturer or consumer, but a positive loss to both. On the other hand, the wool-growing interest will be ruined by the competition of Australia, New Zealand, and the South American State.”
  6. Would you impose specific or ad-valorem duties on steel rails, wool, woolen cloths? Give your reasons. What has been the practice in imposing duties on these articles since 1860?

Final. June, 1886.

_________________

1886-87.
POLITICAL ECONOMY 6
[End-Year]

  1. Comment on the historical statements, and on the reasoning from them, in the following extracts:—
    “Such was the state of things [bankruptcy and ruin the most complete] at the date of the passage of the tariff act of 1842. Scarcely had it become a law, when confidence began to reappear and commerce to revive—the first steps toward the restoration of the whole country, in the briefest period, to a state of prosperity the like of which had never before been known. Seeing that these remarkable facts were totally opposed to the free-trade theory, the author was led to study the phenomena presented in the free-trade period from 1817 to 1824, and in the protective one which commenced in 1825 and ended in 1834,–the one terminating in bankruptcy and ruin similar to that which exhibited itself in 1842, and the other giving to the country a state of prosperity such as had again been realized in 1846….The more he studied these facts, the more did he become satisfied that the free-trade theory embodied some great error.” H.C. Carey, Preface to the Principles of Social Science.
  2. It has been said that protective duties cause the price of the protected articles to fall; and such an effect is said to have been produced on the prices of cotton cloth after 1816, of copper after 1869, and of steel rails after 1870. Comment on the principle, and on its application in these three cases.
  3. “This ill-understood and much reviled principle [the minimum principle] appears to me to be a just, proper, effective, and strictly philosophical mode of laying protective duties. It is exactly conformable, as I think, to the soundest and most accurate principles of political economy. It is, in the most rigid sense, what all such enactments so far as practical be ought to be: that is to say, a mode of laying a specific duty. It lays the import exactly where it will do good and leaves the rest free. It is an intelligent, discerning, discriminating principle, no a blind, headlong, generalizing, uncalculating operation….The minimum principle, however, was overthrown by the law of 1832, and that law, as it came from the House, and as it finally passed, substituted a general and universal ad valorem duty of fifty per cent.” Webster, Speech in the Senate, 1836.
    What were the duties to which Webster refers in this passage? And what should you say to his comments on them?
  4. Explain carefully what is the fundamental proposition in Walker’s Treasury Report of 1845, and discuss its soundness as a principle of tariff reform.
  5. Explain the present system of duties on woolen cloths, stating briefly its history; and say something as to its effects.
  6. It has been said that high duties should be levied on manufactured articles and low duties on raw materials, because raw materials, being more bulky, require much shipping to transport them, and their free admission would give increased employment to American vessels. Assuming that the materials would fact be carried in American vessels, should you say the argument was a sound one?
  7. How can you explain the fact that, while the manufacture of cotton cloths has been little, if at all, dependent on protection, the heavy duties on silk piece-goods have not prevented a continuous large importation?
  8. It has been proposed to admit sugar from Cuba duty free, by a reciprocity treaty. Should you be in favor of such a measure?
  9. Discuss on of the following subjects. (Those who have prepared special reports on any one of these subjects are not to select that one for discussion.)
    1. The financial working of the tariff act of 1846.
    2. Proposed tariff legislation since 1883
    3. The circumstances under which the tariff act of 1833 was passed.

Final. 1887.

_________________

1887-88.
POLITICAL ECONOMY 6
[End-Year]

  1. “I will not argue the question whether, looking to the policy indicated by the laws of 1789, 1817, 1824, 1828, 1832, and 1842, there has been ground for the industrious and enterprising people of the United States, engaged in home pursuits, to expect government protection for internal industry. The question is, do these laws, or do they not, from 1789 to the present time, constantly show and maintain a purpose, a policy, which might naturally induce men to invest property in manufactures, and to commit themselves to those pursuits in life? Without lengthened argument, I shall take this for granted.”—Webster, Speech of 1846.
    Was Webster justified in taking so much for granted?
  2. Compare the treatment of the bearing of protective duties on wages in Hamilton’s Report on Manufactures with the treatment of the same topic in Walker’s Report of 1846, and give an opinion on the value of the discussion at the hands of both statesmen.
  3. What connection has been alleged to exist, and what connection in fact existed, between tariff legislation and general prosperity in 1837-39, in 1843, and in 1857?
  4. Point out wherein the duties on wool and woolens under the act of 1828 resembled, and wherein they differed from, the duties on the same articles under the act of 1867.
  5. Compare the effect of the duties on cotton goods between 1816 and 1824, with the effect of the duties on the same goods between 1864 and 1883.
  6. Point out wherein Mill’s reasoning as to the effect of an import duty on the terms of an international exchange is different from the export tax theory of 1832.
  7. Explain what conclusions you can draw as to the economic effect of the duties on pig iron between 1870 and 1888, from your knowledge of foreign and domestic prices, duties, domestic production, and imports.
  8. Explain why the duty on imported sugar has not stimulated the production of beet sugar in the United States. Apply a similar explanation to some other industry, not connected with agriculture, in which high duties have had less effect than might have been expected.
  9. Point out wherein the course of the tariff legislation of the United States between 1864 and 1883 was similar to the course of legislation in France between 1815 and 1860, and wherein it was not similar.
  10. “First, there is no sufficient market for our surplus agricultural products except a foreign market, and, in default of this, such surplus will either not be raised, or, if raised, will rot on the ground. Second, the domestic demand for the products of existing furnaces and factories is very far short of the capacity of such furnaces and factories to supply; and, until larger and more extended markets are obtainable, domestic competition will inevitably continue, as now, to reduce profits to a minimum and greatly restrict the extension of the so-called manufacturing industries….Industrial depression, business stagnation, and social discontent in the United States, as a rule, are going to continue and increase until the nation adopts a fiscal and commercial policy more liberal and better suited to the new condition of affairs.”— D.A. Wells, in the North American Review.
    Do you think the remedy of lower import duties will remove the difficulties said to arise from excessive production?

Final, 1888.

_________________

1888-89.
POLITICAL ECONOMY 6
[End-Year]

[Arrange your answers strictly in the order of the questions.]

  1. State the duties on cotton cloths, woolen cloths, pig iron, and coffee, in 1790, 1840, 1850, 1885, noting whether the duties were specific or ad valorem, and what tariff acts were in force at these dates, respectively [Use tabular form if you wish.]
  2. “Beside the protection thrown over the manufacturing interest by Congress during this period (1789-1812), the war which raged in Europe produced a favorable effect. As the United States was a neutral nation, she fattened on the miseries of the European nations, and her commerce increased with astonishing rapidity. Our manufactures flourished from the same cause, though not to a corresponding degree with our commerce”
    Did Congress protect manufactures during this period? Did the wars in Europe have the effect described on our commerce and manufactures?
  3. Wherein were the duties on rolled iron in France, in the first half of this century, similar to those in the United States at the same period? How do you account for the similarity, and what was the effect of the duties in either country?
  4. Why was a compound duty imposed on wool in 1828? Why in 1867? Is such a duty now imposed on wool?
  5. Wherein does the present duty on worsted goods differ from that imposed on woolen goods in 1828? wherein from the present duty on woolens? What has been the effect of the difference between the present rates on woolens and worsteds?
  6. Point out some general features in the tariff act of 1846 which were recommended in Secretary Walker’s Report of the year preceding.
  7. What would be the effect of a treaty with Spain admitting free of duty sugar from Cuba?
  8. Wherein has the effect of the duties of the last twenty-five years been different as to cottons, linens, woolens? Why the differences?
    [Omit one of the following:—]
  9. Mill says that certain conclusions which he reaches as to the effect on foreign countries of import duties, do not hold good as to protective duties. Is there good ground for distinguishing as he does
    [Note: Taussig appears to have pasted questions 10 and 11 below over the last line (or two) of question 9.]
  10. “The only case indeed in which personal aptitudes go for much in the commerce of nations is where the nations concerned occupy different grades in the scale of civilization…In the main it would seem that this cause does not go for very much in international commerce. The principal condition, to which all others are subordinate, must be looked for in that other form of adaptation founded on the special advantages, positive or comparative, offered by particular localities for the prosecution of particular industries.”—Cairnes, Leading Principles.
    Discuss, with reference to the general line of reasoning in this passage, the international trade of the United States in (1) glassware, (2) hardware and cutlery, (3) hemp and flax [take any two].
  11. Comment on the following:—
    “The manufacture of silk goods in the United States at the present time [1882] probably supplies an example of an industry which, though comparatively new, can hardly be said to deserve protection as a young industry. The methods and machinery in use are not essentially different from those of other branches of textile manufactures. No great departure from the usual track of production is necessary in order to make silks….Those artificial obstacles which might temporarily prevent the rise of the industry do not exist; and it may be inferred that, if there are no permanent causes which prevent silks from being made as cheaply in the United States as in foreign countries, the manufacture will be undertaken and carried on without needing any stimulus from protecting duties.”— Taussig, Protection to Young Industries.

Final 1889.

Political Economy 6. Grade Distribution 1888-89, 2d half-year.

Total (32) Senior (16) Junior (14) Other (2)
A 2 2
A- 1
B+ 3 2
B 4 4
B- 1 1
C 1 3 2
D 4
E 2

_________________

1889-90.
POLITICAL ECONOMY 6
[End-Year]

  1. What grounds are there for believing that the restrictive policy of Great Britain did or did not have a considerable effect on the industrial development of the American colonies?
  2. What was the effect of the political situation in 1824 on the tariff act of that year? in 1842 on the act of 1842?
  3. “The tariff of 1846 was passed by a party vote. It followed the strict constructionist theory in aiming at a list of duties sufficient only to provide revenue for the government, without regard to protection.”—Johnston’s American Politics.
    Was the act passed by a party vote? Did it disregard protection? Did it succeed in fixing duties sufficient only to provide revenue?
  4. What basis is there for the assertion that the gold premium, in the years after the civil war, increased the protection given by the import duties?
  5. Under what circumstances was the tariff act of 1864 passed? How long did it remain in force?
  6. Is there any analogy between the effects of the duties on cotton goods after 1816 and those on steel rails after 1870?
  7. Wherein would there probably be differences in the effects of reciprocity treaties (1) with Canada, admitting coal free; (2) with Great Britain, admitting iron free; (3) with Brazil, admitting sugar free?
  8. Apply Gallatin’s test as to the effect of duties on the price of the protected articles, to the present facts in regard to (1) clothing wool, (2) silks.
  9. On what grounds is the removal of the duty on pig iron more or less desirable than that of the duty on sugar?
  10. Is it a strong objection to ad valorem duties that they depend on foreign prices and that therefore the duties are fixed by foreigners? Is it a strong objection to specific duties that they operate unequally?

Final. 1890.

Political Economy 6. Grade Distribution 1889-90, 2d half-year.

Total (27) Senior (17) Junior (9) Other (1)
A 2 1
A- 1
B+ 4
B 6 3 1
B-
C+ 1 1
C 3 3
D- 1
E

 

Source: Harvard University Archives. Examination papers in economics, 1882-1935. Prof. F. W. Taussig.

 

Categories
Harvard Suggested Reading Syllabus

Harvard. History of Tariff Legislation. Taussig, 1888

 

This post provides an extended bibliography and syllabus printed in 1888 for Frank W. Taussig’s course on the history of tariff legislation. In a later post I will provide transcriptions of the final exam questions for this course. This artifact is 28 printed pages long!  Exam for June 1888. Exam for June 1889.

__________________

Course Announcement 1888-89

[Political Economy] 6. History of Tariff Legislation in the United States. Half-course. Tu., Th., at 2, and a third hour at the pleasure of the Instructor (second half-year). Professor Taussig.

Source: Harvard University. Announcements of Courses of Instruction provided by the Faculty of Harvard College for the Academic Year 1888-89. Cambridge, May 1888. pp. 18-19.

__________________

TOPICS AND REFERENCES
IN POLITICAL ECONOMY VI.
HARVARD COLLEGE.

TARIFF LEGISLATION IN THE UNITED STATES.

Cambridge, Mass.
1888.

Published for Members of Harvard University by the Harvard Co-operative Society. For others by A. A. Waterman.

 

[p. 2]

POLITICAL ECONOMY VI.

PART I. IMPORTANT DOCUMENTS AND PAPERS.

  1. Hamilton’s Report on Manufactures.

Read Hamilton’s Works, ed. of 1810, I, 157-196; ed. of 1850, III, 192-223; ed. of 1885, III, 294-335.

Summary of the Report:

  1. The relative productiveness of agriculture and manufactures. Rent, as a sign of the productiveness of agriculture.
  2. Circumstances rendering manufactures productive: (1) division of labor; (2) use of machinery; (3) employment of women and children; (4) promotion of immigration; (5) greater diversity of talent; (6) more various field for enterprise; (7) greater demand for products of the soil, “home market.”
  3. Peculiar circumstances of U. S.: (1) absence of reciprocity; (2) cultivation of land not retarded; (3) force of habit opposes manufactures; (4) improbability of success, from (a) scarcity of labor, (b) dearness of labor, (c) scarcity of capital (remedied by funded debt).
  4. General arguments again: (1) will encouragement of manufactures cause a rise in prices? (2) independence in time of war; (3) charge of transportation saved; (4) no opposition of interest between North and South.

[p. 3]

  1. Means for encouraging manufactures enumerated and discussed; e. g. duties on imports, prohibitions of importation, prohibitions of exportation, bounties (commended, and constitutionality maintained), premiums, drawbacks, encouragement of inventions, etc.
  2. List of industries existing, and recommendations in regard to them.

 

  1. Gallatin’s Memorial of 1831.

Read Gallatin’s Memorial on Free Trade, pp. 1-47; the same passages in Congressional Documents, 1stsession, 22nd Congress, Senate Doc., vol. I, No. 5, pp. 1-30, and in The Banner of the Constitution, vol. III, pp. 97-101.

Summary of the Memorial:

  1. The needed revenue, and the average duty which would secure it.
  2. The general principles of free trade.
  3. Compensating advantages from protection, as the employment of female labor [compare Mill, Political Economy, Book I, ch. V, § 1, note], the stimulus to producing some raw materials, the creation of a home market.
  4. Certain arguments for protection: high wages; that foreign trade stimulates foreign industry; the relation of imports and exports; reciprocity and retaliation; the experience of other countries.
  5. The reduction of prices by domestic competition.
  6. Careful and detailed examination of duties then in force.

 

  1. Walker’s Report of 1845.

Read Report of the Secretary of the Treasury for 1845, Executive Documents, 29th Congress, 1st session, vol. II, No. 6, pp. 3-14. Printed also in Niles’s Register, vol. 69, pp. 233-235.

[p. 4]

  1. Noteworthy principles laid down:
    (a) No duty should be imposed above the lowest rate that will yield the largest revenue. What does this mean? (b) Below this rate discrimination may be made. What sort of discrimination would Walker favor? (c) The maximum rate may be imposed on luxuries. (d) All specific and minimum duties should be abolished.
  2. How far the reasoning and the proposals of the report are consistent with the principles of free trade.
  3. The treatment of the effects of a protective tariff on wages and on profits.
  4. Specific and ad valorem duties. The warehousing system.
  5. The general merit of the Report; the praise it has often received. Report of the Tariff Commission of 1882, pp. 1428-1427.

 

PART II. HISTORY OF TARIFF LEGISLATION.

  1. Period before 1789.

General References: Adam Smith, Wealth of Nations(Rogers’s ed.), II, 156-166. Pitkin, Statistical View, ch. I. In general, read on the period till 1816, H. C. Adams, Taxation in the U. S., 1789-1816.

  1. Policy of England. The Mercantile System.
    (a) The Navigation Laws and the Colonial System. (b) Bounties. (c) Prohibitions. (d) History of the iron manufacture, as a type. Bishop, Hist. Manuf., I, 623-629.
  2. Policy of the Colonies.
    (a) Bounties. Bishop, vol. I, passim. (b) Effect of war of revolution. Non-importation agreements. Bishop, I, 365-396.
  3. Industrial state of the Colonies. How far affected by legislation. H. C. Adams, Taxation, etc., 5-13. Thompson, Social Science, 353.

[p. 5]

  1. Tariff acts of individual States before 1789, g.Pennsylvania act of 1785, Hoyt’s Protection versus Free Trade, Preface, p. xii; Adams, Taxation in U. S., 27.
  2. Scheme of a Federal Impost (5% duty) under the Confederation. The effect of its failure on the formation of the Union. Elliot, Debates, 92-106. Pitkin, Statist. View, 26-29.

 

  1. Tariff Act of 1789.

General References: Hamilton, Life of Hamilton, IV, 2-7. Sumner, Protection in U. S., 21-25. Young, Report on Customs Legist., p. xv.

  1. Debate of 1789.
    (a) Madison’s position. Young, Report, p. vii, viii. Madison, Writings,
  2. I, 466, 468. (b) Protectionist views advanced. (c) General tendency of the debate to look mainly at the revenue.
  3. Act of 1789.
    (a) The preamble. (b) Modelled on 5% scheme of confederation. General 5% duty. (c) Duties of 7½ , 10, 15%, on certain articles. (d) Specific duties on cordage, hemp, nails, steel, etc. Hamilton, Works, II, 55.
  4. A common account of the significance of the act of 1789.
    Blaine, Twenty Years of Congress, I, 182-186.
  5. Tonnage act of 1789.
  6. Revenue Collection act of July 31, 1789.

 

  1. 1789-1816.

General References: Bolles, Fin. Hist., II, 73-87. Taussig, Young Ind., 14-21.

  1. Tariff Acts from 1789 till 1816. Gradual increase of duties. Act of 1804 (Barbary Powers act) as an example. Young’s Report, xxxi, xxxii.

[p. 6]

  1. Industrial history, 1792-1807. Expansion of trade, due largely to wars in Europe. Large imports, especially from England.
  2. Restrictions, 1808-1815. Embargo, 1808-1809. Non-intercourse Act, 1809. War, 1812-1815. Duties doubled during the war. Effect of restrictions on foreign trade; on manufactures.
  3. Public opinion on protection in the earlier part of the period. Madison’s attitude in 1789, Young’s Report, p. viii; his Resolutions of 1794, Annals of Congress, 1794, pp. 155, 209. Jefferson’s feeling in 1787, Notes on Virginia, Works, VIII, 404; his Report on Commerce, in 1793, Works, VII, 637-651. Various Committee Reports of this period in American State Papers, Finance, vol. I.
  4. Public opinion during the period of restriction. Clay’s speech of 1810, Works, I, 195-199 (edition of 1848).

 

  1. Act of 1816.

General References: Taussig, Young Ind., 28-34, 40-44. Sumner, 34-38. Calhoun, Works, II, 163-173. Stat. at Large, III, 310-311.

  1. Great growth of manufactures during the war. Manufacturers ask for aid. Appleton, The power-loom, etc., 12-13.
  2. Madison’s Message, Statesman’s Man., I, 331. Dallas’s Report, Am. St. P. Finance, III, 87-91.
  3. Provisions of the act. General increase of duties. Duties on cottons and woollens; on rolled and hammered bar iron. Taussig, Young Ind., 54, 56.
  4. Public opinion not strongly aroused. Attitude of New England, the Middle States, and the South. The act of 1816 marks transition from the period 1789-1815 to the period 1820-32.
  5. The War Argument. Calhoun’s speech of 1816. Holst’s Life of Calhoun, pp. 27-37.

[p. 7]

  1. The Protective Movement after 1819. The Act of 1824.

General References: Taussig, Young Industries, 21-28, 33-40, 43-48, and in Political Science Quarterly, vol. III, March, 1888; Webster, Works, 96-106. Stat. at Large, IV, 25.

  1. The years 1816-19. Inflated prices; large imports; land speculations; reckless banking. Crisis of 1819. Effect on agriculture; on manufactures. Hildreth, Banking, 64-78; Gouge, Hist. of Paper Money, 55-127.
  2. Protective movement after 1819. Agitation for Protection. M. Carey’s pamphlets, Appeal to Common Sense (1822), The Crisis (1823), etc. Niles’s Register.
  3. Tariff acts of 1818. at Large, III, 460, 461.
    Tariff bills of 1820, 1821, 1822.
    Attitude of the Middle and Western States; of New England; of the South. Question of constitutionality raised.
  4. Situation in 1824. Candidates for the presidency: Clay, Crawford, Jackson, Adams. Jackson’s Letter to Coleman. Parton, Life of Jackson, III, 34-36.
  5. Act of 1824. Its history in Congress. Attitude of Massachusetts. The measure acceptable chiefly to the West and Middle States.
    General advance of duties on raw materials (hemp, wool, iron), and on manufactures (cottons, woollens, tern cordage).

 

  1. Tariff Act of 1828.

General References: Taussig, in Political Science Quarterly, vol. III (March, 1888); Calhoun, Works, III, 47-51; Stat. at Large, IV, 270.

  1. Woollen Manufacture, 1824-28. Reduction of duty on wool in England in 1824 and 1825.

[p. 8]

  1. Woollens Bill of 1827. . The Minimum Scheme. Bishop, History of Manuf. II, 313. Bill in full, Annals Congr., III, 731.
  2. The Harrisburg Convention (1827). The demand for higher protection extended to other articles than wool and woollens. Niles, XXXII, 388-396.
  3. Political situation, 1827-28. Democratic leaders from North (Jackson men) combine with Southern members. Attitude of Adams’s supporters, especially those from New England.
  4. Act of 1828; “the tariff of abominations.” Duties on wool (note that on cheap wool); minimum system on woollens (note cheap woollens); on molasses, without drawback on rum; on iron, hemp, flax.
  5. Curious votes on this act. Niles, XXXV, 52-57. Slight political effect on the election of 1828.

 

  1. Agitation in the South. The Export Tax Theory.

General References: McDuffie’s speech, in Congr. Debates, vol. VIII, Part III, pp. 3142-3150. Mill, Political Economy, Book V, ch. IV, §§ 5, 6.

  1. Agitation in the South against the tariff, to which all depression is ascribed. Madison’s Private Correspondence, 274-285.
  2. First form of the export tax theory, as stated in 1830: a tax on imports is a tax on exports, and a tax on the South. McDuffie’s speech of 1830, Congr. Deb., vol. VI, pp. 843-847.
  3. Second form of the theory, in McDuffie’s speech of 1832. (See also his report for the Committee on ways and means in 1832, Reports of Committees, 1st sess., 23d Congr., vol. II, no. 279). The theory worked out in the movement of prices.
  4. The connection between slavery and the export tax theory.

[p. 9]

  1. Acceptance of the export tax theory by the South in 1832. Hayne’s speech, Debates, vol. VIII, pp. 86-90. Address of So. Car. Convention, in State Papers on Nullification, p. 62. Calhoun, Works, III, 411; IV, 182.
  2. The theory soon dropped in the South. Similar reasoning sometimes appears at the present time, e. g. N. Y. Nation, Dec. 31, 1885, and Dec. 15, 1887. How far is it sound?

 

  1. 1828-1832.

General References: Sumner, Life of Jackson, 215-223; Clay, Speech of Jan. 11, 1832, Works, I, 586-595. Stat. at Large, IV, 583.

  1. Tariff Acts of 1830. Tea and coffee free. Abominations of 1828 removed in part. Stat.  at Large, IV, 403, 419.
  2. Public Sentiment on the Tariff. Free Trade Convention in Philadelphia, 1831. Gallatin’s Memorial; Adams, Life of Gallatin, 610-642. Protectionist Convention in New York.
  3. The revenue question. Approaching discharge of the public debt.
  4. Various proposals in 1832.
    (a) Administration scheme. Jackson’s Message, Statesman’s Manual, II, 763. Bill prepared by Secretary McLane, Exec. Doc. 1831-32, vol. 5, No. 22. (b) Southern project. McDuffie’s report and bill, House Rep., 1831-32, vol. 2, No. 279. (c) Clay’s high protection scheme. (d) Moderate protection scheme. J. Q. Adams’s report and bill. House Rep. 1831-32, vol 5, No. 481.
  5. The act of 1832, founded on Adams’s scheme. The duties on iron, wool and woollens, cottons, silks, etc.

[p. 10]

  1. Act of 1833.

General References: Sumner’s Life of Jackson, 281-291. Clay’s speech of Feb. 12, 1833, Works, II, 106-121. Bolles, II, 422-431. Stat. at Large, IV, 629.

  1. Political Situation in 1832-33. Nullification by South Carolina. Re-election of Jackson, and election of a Congress likely to follow his suggestions. The old Congress holds over for the session of 1832-33.
  2. Verplanck bill, supported by the administration. Congr. Debates, vol. IX, p. 958. Its passage found to be impossible.
  3. Clay’s Compromise Scheme. The administration party and the South (Calhoun); the protectionists. Supposed “secret history” of the compromise. Benton’s Thirty Years’ View, I, 342-344. Clay’s speech in 1837, Congr. Debates, XIII, 969-970; Appleton’s speech in 1842, Congr. Globe, X, (Appendix), p. 575.
    The public lands bill fails because of a pocket veto. Amer. Ann. Register, 1832-33, pp. 182-185.
  4. Act of 1833. (a) Gradual reduction of duties. American Almanacfor 1834, p. 138. (b) Treatment of specific duties. U. S. Doc, 1833-34, Exec. Doc, vol I, No. 43. (c) Horizontal rate of 20 per cent. Its general policy. Webster, Works, IV, 258-261. (d) Did the act impose any duties after 1842? Decision of the Supreme Court in Aldridge vs. Williams, 3 Howard, 9.

 

  1. 1833-1842. Tariff of 1842.

General References: Hoist, Constitutional History, II., 451-463. Bolles, II, 426-431, 440-448. Statutes at Large, V, 548.
On the period between 1830 and 1860, see, in general, Taussig, in Quarterly Journal of Economics, April, 1888.

[p. 11]

  1. Economic events of 1833-42. The bank troubles, the crises of 1837 and 1839, the depression of 1839-41. These events sometimes said to be connected with the changes in duties. Carey, Social Science, II, 225; Stebbins, Protectionist Manual, 182.
  2. Operation of the act of 1833. (a) Any effect on manufacturing industries? (b) Accumulation of revenue due to the peculiar features of the act? (c) Attempts to modify it. Woodbury’s Treasury Report of 1835. Bill passed by the Senate in 1837, Congr. Debates, XIII, 939; a similar bill in the House. No proposals in 1837-41. (d) Tariff act of 1841, Stat. at Large, V, 463.
  3. Financial situation in 1842. Political situation. The Whigs and the tariff; Tyler’s position. Effect of these complications on the details of the act.
  4. Provisions of the act. Credits on duties abolished; but no warehousing system.
  5. Debates in 1842. The labor argument. The violation of the compromise of 1833. Not a word as to nullification. Prominence of the iron industry.
  6. Revival of trade in 1843-41. How far this was connected with the passage of the act of 1842.

 

  1. Tariffs of 1846 and 1867.

General References: Hoist, Const. History, II, 529-535, III, 277-280; Webster, Works, V, 225-235; Stat. at Large, IX, 42; XI, 192.

  1. Political situation. Campaign of 1844. Session of 1845-46, and passage of the act of 1846. Allegations of British Gold.
  2. Provisions of the act of 1846. The schedules; the ad valorem duties; the warehousing system. How far did it follow Secretary Walker’s recommendations? How far was it a free trade measure?

[p. 12]

  1. The debates on the act of 1846. The wages argument in the speeches of Hunt, Congr. Globe, 1845-46, Appendix, p. 967, and of Winthrop, ibid., 972-973. Its treatment by Webster.
  2. Financial operation of the act of 1846. Working of the ad valorem duties. Speech of Brooks, Congr. Globe, XXIV, 809-812 (1852). Report of the Secretary of the Treasury for 1853, pp. 62, 104.
  3. Act of 1857. The bill, as originally passed by the House, much amended in the Senate. The changes in duty made by it.
  4. Slavery and the tariff. Attempts by the Whigs to substitute the tariff for slavery as the decisive issue in politics.

 

  1. Economic History, 1840-60.

General References: Grosvenor, Does Protection Protect, 146-150, 223-229.— On manufacturing industries, see, in general, the introduction to the volume on manufactures of the census of 1860.

  1. General prosperity during this period. Can it be ascribed to the tariff acts of 1846 and 1857? — The crisis of 1857.
  2. International trade, and the growth of exports and imports. James, Amerikanischer Zoll-tarif, 49-73; International Review, XI., 450-462. Grosvenor, 50-53.
  3. Iron Manufacture. Uncertainty of the statistics. Course of production, and extent of importation. Anthracite and charcoal iron. Hewitt on Statistics of Iron, 24-32; Statistics in Hewitt’s A Century of Mining in the United States, Appendix, and in the Reports of the American Iron and Steel Association.
  4. The Cotton Manufacture. Batchelder, in Hunt’s Merchant’s Magazine, XLV, 14-16. The domestic consumption of raw cotton; statistics in Quarterly Reports of the Bureau of Statistics, No. 3, 1885-86, p. 60, in Reports of U. S. Comm. to Paris Exhibition of 1867, VI, 30-35, and in Hunt’s Merchant’s Magazine, XLV, 11.

[13]

  1. The Woollen Manufacture. Census of 1860, as above; Special Report of the Bureau of Statistics on the Manufacture of Wool (1887).
  2. The range and extent of manufacturing industry in 1860.

 

  1. The Morrill Tariff. Duties During the War.

General References: Wells, in Cobden Club Essays, Second Series, pp. 473-481.
On the history of legislation between 1860 and 1883, read Taussig, History of the Present Tariff.

  1. The state of the revenue in 1860. The political situation. The Republicans in control of the House in the 36th The tariff bill passed in the House in 1860, in the Senate in 1861.
  2. Provisions of the Morrill tariff act of 1861. Specific substituted for ad valorem duties. The rates on iron, wool and woollens, cottons, etc.
    How far the act was protectionist. Attitude of the manufacturers, especially on the wool and woollen duties.
  3. Financial needs of the civil war. General character of the war legislation. Acts of August and December, 1861, imposing direct tax, and raising revenue duties. Stat. at Large, XII, 292, 330.
  4. Internal Revenue act of 1862. Excise taxes in general at 3% on the value. Young’s Tariff Legisl., p. 126. Corresponding increase in import duties in the tariff act of 1862. Stat. at Large, XII, 433, 543.
  5. Tax and Tariff acts of 1864. Act authorizing $400,000,000 loan. Three-fold object of the tariff act: revenue, compensation of internal taxes and protection. Brief consideration of the bill in Congress. Its importance in financial and economic history. Stat. at Large, XIII. 202, 223.

[p. 14]

  1. Reduction of Duties, 1864-86.

General References: Taussig, Present Tariff, pp. 17-39, 88-101. Perry in Quarterly Journal of Economics, vol. II, pp. 69-78 (Oct., 1887).

  1. Reduction and abolition of the internal taxes, 1865-72. Further changes in 1883. The present internal tax system.
  2. Attempts to reduce duties. The state of opinion on the tariff immediately after the war. The bill of 1867, passed in the Senate, lost in the House.
  3. Act of 1870. Its object, a reduction of revenue. The revenue duties lowered; also a few protective duties, e. g. on pig iron. Young’s Tariff Legisl., p. 167.
  4. Act of 1872. Pressure for a reduction of duties. The protectionist tactics. A general 10 per cent. reduction and a lowering of duties on certain raw materials, substituted for a detailed revision.
    Abolition of duties on tea and coffee in 1872.
    In 1875, repeal of the 10 per cent. reduction, with an increase in the sugar duty, and in the internal tax on spirits.
  5. The Morrison bill of 1876; in Congr. Record, 1875-76, p. 3321. The Wood bill of 1878, Congr. Record, 1877-78, p. 2398; N. Y. Nation, vol. XXVI. pp. 89, 220, 225, 380.
  6. The act of 1883. Tariff Commission act of 1882. The report and recommendations of the Commission. The tariff bill adopted by both houses, on the report of a conference committee. Nature of the reductions in the act of 1883. Report of the Tariff Commission of 1882. Compare Quarterly Reports of the Bureau of Statistics, No. 2. 1886-87, p. 364.
  7. Unsuccessful attempts at legislation, 1883-1887.

[p. 15]

  1. Increase of Duties, 1864-83.

General References: Taussig, Present Tariff, pp. 40-88.

  1. Wool and woollens. Act of 1867. Convention of wool growers and manufacturers, and agreement by them on a tariff scheme. The compensating principle; mixed specific and ad valorem duties. Large increase of duties.
  2. Copper act of 1869. Increase of duties. Character of the act. Vetoed by President Johnson, but passed over the veto.
  3. Act of 1870, while reducing some duties, raises others, e. g. on steel rails, nickel, flax, etc.
  4. Act of 1883. General character of the advance of duties under it.

 

PART III. EFFECT OF TARIFF LEGISLATION SINCE 1860.

Convenient general sources of information are:

Report of the Special Commissioner of the Revenue (D. A. Wells) for 1866, 1867, 1868 and 1869.
Report of the Tariff Commission of 1882, House Misc. Doc, 47th Congr., 2d sess., Doc. No. 6, (referred to in the following pages as Tariff Commission Report.)
Arguments made before the Committee of Ways and Means on the Morrison tariff bill of 1884, House Rep., 48th Congr., 1st session (referred to as Arguments of 1884).
Statements to the Committee of Ways and Means on the Morrison tariff bill of 1886. House Rep., 49thCongr., 1st session (referred to as Statements of 1886).

[p. 16]

Report of the Secretary of the Treasury on the Revision of the Tariff, with accompanying documents. House Exec. Documents, 40th Congr., 1st session (referred to as a Report on Revision, 1886).
Report on the Existing Tariff on Imports, and the Free List, with comparative tables of present and past tariff. Senate Rep., 48th Congr., 1st sess., Rep. No. 12. (referred to as Tariff Compilation, 1884.)
Imports and Duties; a compilation showing the imports and duties on all articles from 1867 to 1883. House Misc. Doc, 48th Congr., 1st sess., No. 49.

 

  1. Iron, 1860-1885.

General References: Mineral Resources of U. S. (1886), 11-23. Wells, Practical Economics, 85-95. Tariff Commission Report, 2010-2022.
Statistics of production, imports, domestic prices, etc., are in the Reports of the American Iron and Steel Association.

  1. Iron Ore. Distance of the iron mines from the coal centres; the ores of Lake Superior. Imports of ore, chiefly from Spain, Elba, and Cuba. Swank, Production and Characteristics of Iron Ore in the United States; Mineral Resources, 39-103.
  2. Pig Iron. Increase of production since 1860. The two “boom ” periods, 1870-72, and 1879-81. Present locality of production: (a) district east of the Alleghanies, anthracite iron; (b) central district, Pennsylvania and Ohio; (c) Western and Southern States. Imports of Pig Iron; regular continuance of the imports of Scotch iron, and its explanation. Fluctuating import of other iron. Character of the pig-iron industry before 1873. J. S. Newberry, in International Review, I, 768-780. Growing production in the South.

[p. 17]

  1. Prices of pig iron, in England and in the United States. Difficulty of making comparisons. The effect of the tariff on prices; is the price in the United States made higher to the full extent of the duty? Probable consequences of removing the duty. (American prices given in Reports of the American Iron and Steel Association; English prices in Reports of British Iron Trade Association, and in the London Economist.)
  2. Bar Iron. Production; Imports; Duty. Difference between the cost of converting pig iron into manufactured iron in England and the United States.
    The manufacture of hardware. Exports of hardware.
  3. Bessemer steel. History of its manufacture in the United States. The product and import. How far prices have been affected by the tariff. Taussig, Present Tariff, p. 107. Swank, Iron in All Ages, ch. 38. Schoenhof, Destructive Influence of the Tariff, ch. 7.
  4. The controversy as to cotton ties. Tariff Comm. Report, pp. 2040, seq. The duty on tin plates. Report on Revision, 1886, pp. 383-392.
  5. The wages question in the iron trade. Mixture of skilled and unskilled workmen. The Amalgamated Iron and Steel Association, and its possible influence on the wages of skilled workmen. Discussion of the wages of unskilled laborers in (Wells’s) Census Revelations, etc.

 

  1. Cottons.

General References: Report on Cotton Manufacture in Census of 1880, pp. 5-15: Wells, Practical Economics, 81-85.
On the duties on cottons in general, see the statements in Arguments of 1884, pp. 123-181.

  1. Extent and Importance of the Cotton Manufacture.

[18]

  1. Domestic production of cheap goods. Is it desirable to check the importation of foreign cottons of lower price and poorer quality? Is it desirable to prevent foreign manufacturers from making occasional sales, at abnormally low prices, to get rid of surplus stocks?
  2. Imports of cottons. Character of the grades imported. Increase in the duty in 1883.
  3. Reasons why coarse cottons are manufactured successfully. Why the failure to manufacture finer qualities.
    Can popular education, no standing army, general intelligence, be adduced as causes enabling manufacturers in the United States to compete on equal terms with foreign manufacturers?

 

  1. Silks.

General References: Wyckoff, Silk Manufacture, 42-51; Schoenhof, Industrial Situation, ch. Ill, V, VI.

  1. Various attempts to encourage the production of raw silk in the United States. The colonial period; bounties in Georgia. Bishop, History of Manufactures, I, 358. The morus multicaulis speculation of 1830-40; see The Silk Culture in the United States (1844). Raw silk now admitted free. Whence it comes.
  2. History of the silk manufacture. Sewing-silk. State of the industry in 1860. Census Report of 1860, pp. 94-105.
    Great growth since 1865, especially since 1870, under the influence of high duties. How far were the duties originally intended to have this effect?
  3. Continued imports of silks. Reasons adduced why imports continue, in spite of the high duties: (a) adulteration of foreign silks; (b) the lower wages in European countries; (c) the difficulty arising from the nature of raw silk, of applying machinery in its manufacture in the same degree as in other textile industries. Wyckoff’s Silk Goods of America, pp. 7-40.

[p. 19]

  1. Administrative difficulties. The temptations to fraud under the high ad valorem duty. The consignment system. Reasons why it is difficult to substitute specific for ad valorem duties. Tariff Commission Report, 1048-1052, 1605-1613, 2165-2174.

 

  1. Wool and Woollens.

General References: Bulletin Wool Manufacturers, XV, 210-226: the same passage in Report on Revision, 299-313. Schoenhof, Destructive Influence, 17-35, Industrial Situation, 23-31. Taussig, Present Tariff, 53-64.
Statistics and general information are given in the Special Report of the Bureau of Statistics on Wool and Manufactures of Wool, 1887.

  1. Production of wool. Increase since 1860; transfer to the West. Bulletin Wool Mf., XIII, 102-106; Wool Report of 1887, p. 162. Meaning of this change.
    Character of American wool. Influence of climate and other physical causes.
  2. Imports of wool. What grades are imported, and why. Carpet-wool; Tariff Commission Report, 2414, 2415.
  3. Effect of the duties on wool. (a) Immediate effect of the act of 1867. Report of Special Commissioner of Revenue, 1869, p. 93; Bulletin of Wool Mf., II, pp. 2-34. (b) Effect on the prices of wool at home and abroad. Tables of prices in Wool Report of 1887, in London Economist, in Soetbeer’s Materials on the Silver Question, and in commercial circulars. (c) Temptation to fraudulent undervaluation of wool under the minimum duties. Osborn, The Administration and Undervaluation Frauds, 58, 78. Tariff Commission Report, 468. Report on Revision, 242.
  4. Production of woollens. Stimulus given by the war to the manufacture of woollens; depression after the war. Circumstances under which the act of 1867 was passed. The state of the manufacture in 1867-73. The census returns of 1870 and 1880. Character of the goods chiefly made, and their quality.

 

[p. 20]

  1. Imports of woollen goods. Their steady continuance.
    Character of the goods imported.
  2. Effect of the duties on woollens.
    (a) Effect on the consumer. How far an increased price is caused. Need of distinguishing between the effect of protection to wool and that of protection to woollens. Difference between the coarser and the finer qualities of woollen goods.
    (b) Effect on the manufacturer. Causes of the comparatively limited range of the wool manufacture.
    (c) Administrative difficulties. The mixture of specific and ad valorem duties. The minimum duties on dress goods, blankets, etc.
    (d) Change in the method of manufacture in recent years, and its unexpected effect on the working of the tariff. Report of the Secretary of the Treasury, 1887, p. 19; Wool Report of 1887, p. XXIV: Bulletin Wool Mf., XIV, 293-311.

 

  1. Sugar.

General References: D. A. Wells, in Princeton Review, VI, 319-335.

  1. The production and imports of sugar. Extent of the domestic product. Whence the imports come. Statistical Abstract, 1886: Reports on Commerce and Navigation.
  2. The duty on sugar before 1883, on the Dutch Standard. The Treasury rulings, and the decision of the Supreme Court. Change to the Polariscope test in the act of 1883.
  3. Free admission, by treaty, of sugar from the Hawaiian Islands, and its effect. Tariff Commission Report, 695-697; Statements of 1886, pp. 11-22. Anonymous pamphlet on The Hawaiian Treaty; statistics in Quart. Rep. Bureau of Statistics, No. 2, 1885-86.

[p. 21]

  1. Financial and economic aspect of the sugar duty. Proposals to reduce it, and to abolish it with or without compensation to Louisiana planters.
  2. Attempts to stimulate beet culture in the United States. Report of the Department of Agriculture on the Culture of the Sugar Beet, 1880 (note pp. 167-170). The experiment in making beet-sugar in California. Hilgard, in Overland Monthly, December, 1886.
    Reasons why beet sugar has not been made in the United States.
  3. The taxation of sugar in Europe. (a) Gradual abolition of duties in England. (b) On the continent, excise taxes on beet sugar. History of the beet sugar industry, and its connection with protection. Kaufmann, Die Zucker Industrie. (c) The system of bounties on exports, and its recent development. N.Y. Nation, XLII, 420; Wells, in Popular Science Monthly, Jan., 1888.

 

  1. Certain Raw Materials.

In general, consult the document referred to on pp. 15, 16.

  1. Character and extent of the imports and of the domestic production. The effect of the duty. Mineral Resources, 224-234, 242; Report of the Special Commissioner of Revenue for 1869, p. lxxxix; Arguments p. of 1884, p. 361; Statements of 1886, p. 187.
  2. Tariff Commission Report, pp. 955, 1015, 1599, 2379. Has the duty had the effect of promoting a dangerous destruction of forests?
  3. Hemp and Flax; Linens. The manner in which hemp and flax are raised, and the reasons why their importation continues. Report of the U. S. Commission of 1865, Senate Exec. Doc, 38th Congr., 2nd sess., No. 35; the Reports of the Flax and Hemp Spinners and Growers’ Association; Tariff Commission Report, 1452-1456.
    Tariff Commission Report, 345, 1145, 1452.
    Jute. Tariff Commission Report, 345, 1145, 1452.

[p. 22]

  1. Miscellaneous Articles.

 

  1. Copper. Circumstances under which the act of 1869 was passed. Taussig, Present Tariff, 65, 106. Conditions of production; no imports, considerable exports. Mineral Resources, 109-139; Tariff Commission Report, 2177, 2555.
  2. Tariff Commission Report, 201, 219; Wells, Practical Economics, 124; Mineral Resources, 169-173.
  3. Tariff Commission Report, 2591-2597; Mineral Resources, 160.
  4. Taussig, Present Tariff, 70; Tariff Commission Report, 227, 1553, 1648; H. M. Seely, The Marble Border of Western New England, in Proc. Middlebury Hist. Soc., vol. 1, part II, pp. 24-52.
  5. Congr. Globe, 1859-60, pp. 1020-1022; Tariff Commission Report, 227, 726, 941.
  6. Tariff Commission Report (Index).
  7. Tariff Commission Report, 613, 743, 753, 759, 2399; Arguments of 1884, 297.
  8. Arguments of 1884, 245-296. Census of 1880, vol. II., Report on Glass.

 

  1. General Discussions.

General References: Springer in North Amer. Review, vol. 136, pp. 571-580; reviewed in Bulletin Wool Manuf., vol. 13, pp. 199-211. Wells, Practical Economics, 98-116.

  1. Attempts to measure the effect of the protective system. Exaggerated estimates sometimes made. Mongredien, The Western Farmer in America; reviewed by Jonathan B. Wise. Difficulties of reaching numerical results.
  2. The enhancement of the price of articles made at home, in consequence of duties, takes place at present chiefly with raw materials. Is such an effect on raw materials more harmful than a similar effect on manufactured articles?
    Manufactures on which there is a protective tax (silks, earthenware). On very large classes of manufactures the duty is either a revenue duty (finer linen goods), or only nominal (ordinary cottons).

[p. 23]

  1. The effect of the protective system on general prosperity. The depression of 1874-78 and of 1883-85 often ascribed to it. How far such statements can be supported.
  2. Does the protective system tend to the accumulation of large fortunes, and to a spirit of communism? Rathbone, Protection and Communism (1884).
  3. The future of the United States as a manufacturing country. The prospects of New England.
  4. How far is it possible to trace the effect of protective duties?

 

  1. Specific and Ad Valorem Duties.

General References: James, Amerikanischer Zoll-tariff, 36-48; Webster, Works, V, 170-186; Tariff Commission Report, 1090-1092.

  1. Objections urged against specific duties: (a) that they are unequal, and bear more heavily on the poor than on the rich; (b) that they remain the same, though the price of the dutiable articles may vary greatly (steel rails in 1880-85); (c) that Congress is not capable of fixing them intelligently; (d) that their real incidence and effect are apt to be concealed.
  2. Objections to ad valorem duties: (a) the danger of under-valuation and fraud. The present duty on silks and its effect. Tariff Commission Report, 2469-2475. (b) The difficulties of administration.
  3. Mixed specific and ad valorem duties, as on woollens and on marble.
    Minimum duties, as in the tariff of 1828 on woollens, and at present on carpet wool, blankets, etc.

[p. 24]

  1. Distinctions to be made: (a) whether the article is homogeneous (pig-iron), or varies greatly in quality and character (silks). (b) whether duties are collected along a land frontier (Germany), or in comparatively few seaports (England; United States). (c) The character of the administrative system. Civil service reform.
  2. Tendency of free-traders to favor ad valorem duties, of protectionists to favor specific duties. Explanation of these tendencies.
  3. Practise in the United States. Judicious system until 1816; both specific and ad valorem duties. James, 29-36. Operation of the ad valorem system of the act of 1846. Strong tendency toward specific duties since 1861. Difficulties arise as duties go higher after 1816.
  4. Practise in foreign countries. None other than specific duties in England, Germany, and France.

 

  1. Collection of Duties.

General References: Bolles, Financial History, III., 489-523; Goodnow, in Political Science Quarterly, I., 36-44.

  1. History of the Revenue Collection Laws. Report on Revision, pp. 8-27.
    Act of 1789, dividing off customs districts, and establishing the offices of collector, surveyor, and naval officer. Statat Large, I. 36-37.
    Provisions of the acts of 1789, for ascertaining dutiable value; maintained until 1832.
  2. Acts of 1799 and 1818, establishing the moiety system; in 1799, as to penalties and forfeitures, in 1818 as to the extra duty of 50 per cent. on under-valued goods. Stat. at Large, I. 697; III. 437.
    Difficulties experienced under the moiety system. The Phelps-Dodge case; abolition of the moiety system in 1874. Bolles, as above; Stat. at Large, XVIII, 391.

[p. 25]

  1. Various devices for securing correct assessment of ad valorem duties. Appraisers appointed in 1818; additional appraisers in 1830, and in recent years. Invoices required to be sworn to before U. S. consuls in 1863.  Stat. at Large, XII, 737. Special agents authorized in 1878. The “fraud roll” authorized in 1879. Report of Secretary of Treasury, 1885, Appendix, p. 38.
  2. Frequent ambiguity in the tariff laws. The similitude clauses. The packing clause of the act of 1883 (construed in 6 Supreme Ct. Reporter, 462).
  3. Credit on duties allowed from 1789 till 1842. The compromise act of 1833 requires duties.to be paid in cash after June 30, 1842; the act of 1842 also requires cash duties. The act of 1846 retains cash duties, but establishes a general warehousing system.

 

  1. English Tariff History.

General References: Morley, Life of Cobden, ch. IX. Noble, Fiscal Legislation, ch. II., III. Bigelow, Tariff Question, pp. 1-17.

  1. The protectionist system of the 18th century. Pitt’s attempt at reform in 1876-87. Levi, British Commerce, 52-55.
  2. Huskisson’s measures in 1822-26. Modification of the navigation laws. Reduction of import duties on raw materials (wool, silk, metals) and on manufactures (woollens, cottons, silks).
  3. The corn law agitation. The corn law of 1815; the sliding scale of 1828. Anti-Corn-Law League formed in 1838.
    Character of the agitation carried on by the League. Cobden as an agitator. The causes which made certain the ultimate success of the League. Attitude of the manufacturers.
    New sliding scale in 1842. The distress of 1845-46. Repeal of the cornlaw, 1846.

[p. 26]

  1. The four great measures of Peel and Gladstone in 1842, 1846, 1853, and 1860. These measures largely of a fiscal character. Their fiscal and administrative qualities, as compared with tariff acts in the U. S. How far they are separable from the corn-law agitation.
    The present English tariff. Whitaker’s Almanac.
  2. The discussion of English tariff history in the United States. Was protection retained in England until it could he given up without a sacrifice?
    How far the supremacy of England as a manufacturing country is due to the fostering influence of the protective system of the 18th century.
    How far the growth of England since 1846 has been due to free trade.
  3. The connection between the repeal of the corn-laws, and the tariff of 1846 in the United States. Walker’s Report of 1845; p. 11; Webster, Works, V. 231.

 

  1. French Tariff History.

General References: Amé, Tarifs de Douane, vol. I, pp. 34-69. Morley, Life of Cobden, ch XXIX.

  1. Colbert, and the restrictive system of the 17thand 18th Clément, Système Protecteur.
  2. The commercial treaty with England in 1786 breaks with the restrictive system. Amé, I, 25.
    The French assembly establishes a moderate general tariff in 1791.
  3. Outbreak of the Revolutionary Wars. Re-establishment of the prohibitive system. Napoleon and the Continental system.
  4. The situation in 1814-15. Unsuccessful attempt to get rid of the prohibitive system. Gradual extension of the system, and its continuance until 1860. Efforts to get rid of it under the Restoration and under Louis Philippe. Why these efforts failed.
    Possible analogy with the experience of the United States after the civil war.

[p. 27]

  1. The government of the second empire is impelled to undertake reforms. Proposed revision of 1856.
    Commercial treaty of 1860. Its negotiation through Cobden and Chevalier. Its provisions; preparation of the tariff treaty.
    France concludes treaties with other countries than England. General adoption of the treaty system by European countries. Journal of Statistical Society, vol. 40, p. 1.
  2. The effect of the prohibitive system in France. Improvements in production retarded (iron, textiles)? The growth of the international trade after the commercial treaties. The continued advance of France as a manufacturing country under the system of moderate duties.
  3. Current toward protection in recent years. The treaty with England terminated, and the general tariff of 1882 enacted. Change from ad valorem to specific duties. Bounties on shipping and on sugar. Increase of duty on wheat in 1887. Guyot, The French Corn Laws. For the duties now in effect in France, see Report on Revision, pp. 593, seq.

 

  1. German Tariff History.

General References: Article on Zoll-verein in McCulloch’s Dictionary of Commerce, new ed. Wells, in Popular Science Monthly, Jan. 1888.

  1. Germany in the 18th century: (a) the country split into numerous petty States; (b) general application of the mercantile system. The policy of Frederick the Great a typical instance. Schmoller, Jahrbuch für Gesetzgebung, vol. VIII.
  2. Reform in Prussia in 1818. Trade between the different provinces of Prussia made free; a moderate uniform tariff established. In general, see Jahrbuch für Nat. Oek., Supplement VII. (1881); Worms, L’Allemagne Economique.

[p. 28]

  1. The agitation for a customs union. Frederic List. Gradual formation of unions between different States. Final formation of the Zoll-verein in 1834. The Zoll-verein tariff based on the Prussian tariff of 1818.
  2. The later history of the Zoll-verein. Contest in regard to the attitude of Austria; commercial treaty of 1853 with Austria. Contest between free-traders and protectionists. Reduction of duties through the treaty negotiated by Prussia with France in 1862. Zoll-verein renewed in 1865 on basis of French treaty. Treaties with other countries.
  3. In recent years a current toward protective duties and bounties in Continental Europe. The movement for protection begins in Germany after the crisis of 1873, and causes the protective tariff of 1879. Jahrbuch für Nat. Oek., vol. 34, and Supplements V and VI. The agitation for still higher duties on agricultural products.
    The duties under the present German tariff. Report on Revision, 630 seq.
  4. Austria breaks with the prohibitive system by the Zoll-verein treaty, and other treaties.
    Protectionist reaction after 1873. The tariff act of 1882 increases duties. Jahrbuch für Gesetzgebung, VI. 1223-1258.
    Frequent suggestions of a customs-union between the German Empire, Austro-Hungary, and the Balkan States.
  5. The arguments on protection in Germany. List’s argument for protection to young industries. Care for the laboring classes the main argument at present. Joined with it, the argument for compensation to employers for burdens imposed by legislation for social reform. Lexis, in Schönberg’s Handbuch, pp. 1104-1119.

 

Source: Frank W. Taussig.Tariff Legislation in the United States. Topics & References in Political Economy VI, Harvard College.Cambridge, Mass., 1888.

Image Source:  Harvard Class Album 1900.